NeuroSurgery Flashcards

1
Q

A 54-year-old female was taken to an emergency
room after collapsing at work. She was alert and communicative, with a severe headache, photophobia, nuchal rigidity,
and blurry vision. Computed tomography (CT) of the brain
revealed diffuse subarachnoid blood in the basal cisterns,
mild hydrocephalus, and no intraparenchymal hematoma.
Her angiogram is depicted below. What is the clinical Hunt and Hess grade of this patient?
A. Grade I
B. Grade II
C. Grade III
D. Grade IV
E. Grade V

A

A. Grade I
**B. Grade II **
C. Grade III
D. Grade IV
E. Grade V

Patients with
posterior communicating artery (PComA) aneurysms typically present with subarachnoid hemorrhage (SAl-I) and
partial or complete third nerve palsies (ptosis, dilated pupil,
extraocular muscle abnormalities) due to cOIl1pression of the
third nerve by the aneurysm . . Another common presentation
of PComA aneurysms is the development of a third nerve
deficit in the absence of SAl-I. The appearance of an enlarged
pupil with or without involvement of other third nerve ftmctions should be taken as diagnostic of a PComA aneurysm
until proven otherwise. After the aneurysm is clipped, it
should be punctured not only to ensure complete obliteration but also to achieve maximal decompression of the third
nerve. Most patients with third nerve palsies improve within
6 months and frequently sooner. Some PComA aneurysms
will not produce any oculomotor nerve deficit. Special care
must be taken in interpreting the angiograms of these
patients, since the aneurysms often project laterally onto the
medial edge of the temporal lobe rather than in more common posterolateral or downward directions. This is relevant
during operative planning, since early re traction of the
temporal lobe may result in premature aneurysmal rupture.

How well did you know this?
1
Not at all
2
3
4
5
Perfectly
2
Q

A 54-year-old female was taken to an emergency
room after collapsing at work. She was alert and communicative, with a severe headache, photophobia, nuchal rigidity,
and blurry vision. Computed tomography (CT) of the brain
revealed diffuse subarachnoid blood in the basal cisterns,
mild hydrocephalus, and no intraparenchymal hematoma.
Her angiogram is depicted below. Some posterior communicating artery (PComA) aneurysms
do not produce any third nerve deficit. Why should special
attention be given to the angiogram in these cases?

A. If the aneurysm is projecting posterolaterally rather
than in a more common medial position, there is an
increased risk of injuring the perforating vessels from
the PComA during microdissection
B. An aneurysm projecting laterally onto the medial edge
of the temporal lobe argues against premature retraction of the temporal lobe
C. The angiogram may reveal a ventral carotid wall
aneurysm instead of a PComA lesion, which is often
better managed with coiling
D. To look for any other associated aneurysms and/or
vasospasm
E. It may help with surgical planning, as medially projecting lesions are better approached through the carotidoculomotor triangle

A

A. If the aneurysm is projecting posterolaterally rather
than in a more common medial position, there is an
increased risk of injuring the perforating vessels from
the PComA during microdissection
**B. An aneurysm projecting laterally onto the medial edge
of the temporal lobe argues against premature retraction of the temporal lobe **
C. The angiogram may reveal a ventral carotid wall
aneurysm instead of a PComA lesion, which is often
better managed with coiling
D. To look for any other associated aneurysms and/or
vasospasm
E. It may help with surgical planning, as medially projecting lesions are better approached through the carotidoculomotor triangle

Patients with
posterior communicating artery (PComA) aneurysms typically present with subarachnoid hemorrhage (SAl-I) and
partial or complete third nerve palsies (ptosis, dilated pupil,
extraocular muscle abnormalities) due to cOIl1pression of the
third nerve by the aneurysm . . Another common presentation
of PComA aneurysms is the development of a third nerve
deficit in the absence of SAl-I. The appearance of an enlarged
pupil with or without involvement of other third nerve ftmctions should be taken as diagnostic of a PComA aneurysm
until proven otherwise. After the aneurysm is clipped, it
should be punctured not only to ensure complete obliteration but also to achieve maximal decompression of the third
nerve. Most patients with third nerve palsies improve within
6 months and frequently sooner. Some PComA aneurysms
will not produce any oculomotor nerve deficit. Special care
must be taken in interpreting the angiograms of these
patients, since the aneurysms often project laterally onto the
medial edge of the temporal lobe rather than in more common posterolateral or downward directions. This is relevant
during operative planning, since early re traction of the
temporal lobe may result in premature aneurysmal rupture.

How well did you know this?
1
Not at all
2
3
4
5
Perfectly
3
Q

A 54-year-old female was taken to an emergency
room after collapsing at work. She was alert and communicative, with a severe headache, photophobia, nuchal rigidity,
and blurry vision. Computed tomography (CT) of the brain
revealed diffuse subarachnoid blood in the basal cisterns,
mild hydrocephalus, and no intraparenchymal hematoma.
Her angiogram is depicted below.

The patient is taken to the operating room for aneurysm
clipping. Proximal and distal control of the internal carotid
artery is obtained with temporary clip placement prior to
aneurysmal neck dissection. Despite this maneuver, the
aneurysm ruptures during microdissection and significant
bleeding is encountered, which significantly hinders visualization. ‘Vhat preventative maneuver could have been
employed prior to aneurysmal rupture to deCl’ease the
amount of intraoperative bleeding?

A. Blunt surgical microdissection
B. Obtaining proximal control of the internal carotid
artery in the neck
C. ReleaSing the dome of the aneurysm from the temporal
lobe prior to temporary clip placement to prevent traction on the fundus
D. Identifying the distal posterior communicating artery
medial to the internal carotid artery for temporary clip
placement if possible
E. Temporary clip placement on the ophthalmic artery to
prevent retrograde bleeding from the orbit

A

A. Blunt surgical microdissection
B. Obtaining proximal control of the internal carotid
artery in the neck
C. ReleaSing the dome of the aneurysm from the temporal
lobe prior to temporary clip placement to prevent traction on the fundus
**D. Identifying the distal posterior communicating artery
medial to the internal carotid artery for temporary clip
placement if possible **
E. Temporary clip placement on the ophthalmic artery to
prevent retrograde bleeding from the orbit

Patients with
posterior communicating artery (PComA) aneurysms typically present with subarachnoid hemorrhage (SAl-I) and
partial or complete third nerve palsies (ptosis, dilated pupil,
extraocular muscle abnormalities) due to cOIl1pression of the
third nerve by the aneurysm . . Another common presentation
of PComA aneurysms is the development of a third nerve
deficit in the absence of SAl-I. The appearance of an enlarged
pupil with or without involvement of other third nerve ftmctions should be taken as diagnostic of a PComA aneurysm
until proven otherwise. After the aneurysm is clipped, it
should be punctured not only to ensure complete obliteration but also to achieve maximal decompression of the third
nerve. Most patients with third nerve palsies improve within
6 months and frequently sooner. Some PComA aneurysms
will not produce any oculomotor nerve deficit. Special care
must be taken in interpreting the angiograms of these
patients, since the aneurysms often project laterally onto the
medial edge of the temporal lobe rather than in more common posterolateral or downward directions. This is relevant
during operative planning, since early re traction of the
temporal lobe may result in premature aneurysmal rupture.

How well did you know this?
1
Not at all
2
3
4
5
Perfectly
4
Q

A 54-year-old female was taken to an emergency
room after collapsing at work. She was alert and communicative, with a severe headache, photophobia, nuchal rigidity,
and blurry vision. Computed tomography (CT) of the brain
revealed diffuse subarachnoid blood in the basal cisterns,
mild hydrocephalus, and no intraparenchymal hematoma.
Her angiogram is depicted below

Postoperatively, the patient wakes up with contralateral
wealmess, numbness, and homonymous hemianopia. A CT
scan of the brain shows an infarct in the posterior limb of the
internal capsule and in the adjacent white matter (above the
temporal horn of the lateral ventricle). This complication
might possibly have been avoided by

A. Identifying the anterior choroidal artery prior to
aneurysm clipping in order to prevent damage or
incorporation of this vessel into the clip construct
B. Increasing temporary occlusion time to prevent hasty
microdissection
C. Limiting the sylvian fissure dissection to the sphenoidal portion in order to prevent unnecessary
dissection adjacent to PComA artery perforators,
which supply the posterior limb of the internal capsule
D. Obtaining an intraoperative angiogram to confirm
proper clip placement
E. Identifying and preserving the recurrent artery of
Heubner

A

**A. Identifying the anterior choroidal artery prior to
aneurysm clipping in order to prevent damage or
incorporation of this vessel into the clip construct **
B. Increasing temporary occlusion time to prevent hasty
microdissection
C. Limiting the sylvian fissure dissection to the sphenoidal portion in order to prevent unnecessary
dissection adjacent to PComA artery perforators,
which supply the posterior limb of the internal capsule
D. Obtaining an intraoperative angiogram to confirm
proper clip placement
E. Identifying and preserving the recurrent artery of
Heubner

Patients with
posterior communicating artery (PComA) aneurysms typically present with subarachnoid hemorrhage (SAl-I) and
partial or complete third nerve palsies (ptosis, dilated pupil,
extraocular muscle abnormalities) due to cOIl1pression of the
third nerve by the aneurysm . . Another common presentation
of PComA aneurysms is the development of a third nerve
deficit in the absence of SAl-I. The appearance of an enlarged
pupil with or without involvement of other third nerve ftmctions should be taken as diagnostic of a PComA aneurysm
until proven otherwise. After the aneurysm is clipped, it
should be punctured not only to ensure complete obliteration but also to achieve maximal decompression of the third
nerve. Most patients with third nerve palsies improve within
6 months and frequently sooner. Some PComA aneurysms
will not produce any oculomotor nerve deficit. Special care
must be taken in interpreting the angiograms of these
patients, since the aneurysms often project laterally onto the
medial edge of the temporal lobe rather than in more common posterolateral or downward directions. This is relevant
during operative planning, since early re traction of the
temporal lobe may result in premature aneurysmal rupture.

How well did you know this?
1
Not at all
2
3
4
5
Perfectly
5
Q

A 54-year-old female was taken to an emergency
room after collapsing at work. She was alert and communicative, with a severe headache, photophobia, nuchal rigidity,
and blurry vision. Computed tomography (CT) of the brain
revealed diffuse subarachnoid blood in the basal cisterns,
mild hydrocephalus, and no intraparenchymal hematoma.
Her angiogram is depicted below

Postoperatively, the patient sustained damage to the
frontal branch of the facial nerve. What is the most likely
reason for the frontal branch facial nerve injury?

A. The supraorbital nerve was not identified in detaching
the scalp from the supraorbital rim
B. The incision was started less than 1 cm anterior to the
tragus
C. ·There was nerve neuropraxia from postoperative
swelling
D. The nerve in the subgaleal fat pad was injured during
surgical dissection
E. The nerve between the superficial and deep layers of
the temporalis fascia was injured with monopolar
cautery

A

A. The supraorbital nerve was not identified in detaching
the scalp from the supraorbital rim
B. The incision was started less than 1 cm anterior to the
tragus
C. ·There was nerve neuropraxia from postoperative
swelling
**D. The nerve in the subgaleal fat pad was injured during
surgical dissection **
E. The nerve between the superficial and deep layers of
the temporalis fascia was injured with monopolar
cautery

Patients with
posterior communicating artery (PComA) aneurysms typically present with subarachnoid hemorrhage (SAl-I) and
partial or complete third nerve palsies (ptosis, dilated pupil,
extraocular muscle abnormalities) due to cOIl1pression of the
third nerve by the aneurysm . . Another common presentation
of PComA aneurysms is the development of a third nerve
deficit in the absence of SAl-I. The appearance of an enlarged
pupil with or without involvement of other third nerve ftmctions should be taken as diagnostic of a PComA aneurysm
until proven otherwise. After the aneurysm is clipped, it
should be punctured not only to ensure complete obliteration but also to achieve maximal decompression of the third
nerve. Most patients with third nerve palsies improve within
6 months and frequently sooner. Some PComA aneurysms
will not produce any oculomotor nerve deficit. Special care
must be taken in interpreting the angiograms of these
patients, since the aneurysms often project laterally onto the
medial edge of the temporal lobe rather than in more common posterolateral or downward directions. This is relevant
during operative planning, since early re traction of the
temporal lobe may result in premature aneurysmal rupture.

How well did you know this?
1
Not at all
2
3
4
5
Perfectly
6
Q

A 28-year-old male was involved in a motorcycle
accident. About 1 week after being discharged from the
hospital he began experiencing fevers, severe retroorbital
headaches, diplopia, and left eye proptosis, which prompted
a visit to the emergency department. A computed tomography
(CT) scan of the brain showed a ‘ resolving 2- by 3-cm left
frontal contusion underlying a minimally displaced frontal
bone fracture, which was sustained at the time of initial
injury. His erythrocyte sedimentation rate (ESR) and Creactive protein (CRP) were mildly elevated. The angiogram
is depicted below

What is the most likely diagnosis?
A. Superior orbital fissure syndrome
B. Incidental meningioma originating fro111 the medial
aspect of the sphenoid ridge
C. Arterial-venous fistula
D. Occlusion of the internal carotid artery proximal to the
ophthalmic artery ol:igin
E. Cavernous sinus thrombosis

A

A. Superior orbital fissure syndrome
B. Incidental meningioma originating fro111 the medial
aspect of the sphenoid ridge
**C. Arterial-venous fistula **
D. Occlusion of the internal carotid artery proximal to the
ophthalmic artery ol:igin
E. Cavernous sinus thrombosis

Carotid-cavernous fistulas (CCFs) can be
divided into posttraumatic and spontaneous types. They are
direct shunts between the ICA or ECA and cavernous sinus
and usually occur after trauma or spontaneous aneurysmal
rupture. Traumatic CCFs often present in a delayed fashion ;
like spontaneous fistulas, they often present with retro-orbital
199
200 Intensive Neurosurgery Board Review
pain, chemosis, pulsatile proptosis, ocular or cranial bruit,
decreased visual acuity, diplopia, and rarely epistaxis and
subarachnoid hemorrhage. The symptoms depend on the
direction of venous flow and quantity of blood flow through
the fistula. There are four types of CCFs: type A is a direct,
high-flow shunt between the ICA and cavernous sinus (as
in this case), and types B to D are low-flow shunts between
the cavernous sinus and meningeal branches of the internal
carotid artery, external carotid artery, or both, respectively. Approximately 50% of low-flow fistulas spontaneously
thrombose without treatment. The main treatment option
has traditionally included transarterial balloon embolization
through the ICA for type A fistulas, although accessing the
fistula transvenously (Le., inferior petrosal sinus) is also
commonly performed, especially for indirect types B to D.
A direct surgical approach is indicated if transarterial or
transvenous approaches fail. Radiosurgery has been proposed as an option for some of the low-flow fistulas, although
it would not be the best strategy for the high-flow symptomatic fistula seen in this patient. Figure A depicts nearly
complete capture of the blood from the internal carotid
artery, and fistulous drainage primarily from the superior
ophthalmic and superior petrosal veins. Figure B depicts a
later venous run with superior petrosal vein drainage into the
transverse-sigmoid sinus junction as well as some venous
drainage into the superior sagittal sinus (Kaye and Blacl~,
pp. 1132; Greenberg, pp. 811- 812; Youmans, pp. 2341-
2352; Will~ins, pp. 2529- 2535).

How well did you know this?
1
Not at all
2
3
4
5
Perfectly
7
Q

A 28-year-old male was involved in a motorcycle
accident. About 1 week after being discharged from the
hospital he began experiencing fevers, severe retroorbital
headaches, diplopia, and left eye proptosis, which prompted
a visit to the emergency department. A computed tomography
(CT) scan of the brain showed a ‘ resolving 2- by 3-cm left
frontal contusion underlying a minimally displaced frontal
bone fracture, which was sustained at the time of initial
injury. His erythrocyte sedimentation rate (ESR) and Creactive protein (CRP) were mildly elevated. The angiogram
is depicted below

The signs/symptoms of this disease process depend
mostly upon

A. The size and location of the tumor relative to the optic
nerve
B. The direction of venous drainage and rate of blood flow
through the shunt
C. The extent of the inflammatory reaction adjacent to
the cavernous sinus
D. The extent of the inflammatory reaction adjacent to
the superior orbital fissure
E. The extent of collateral flow from the opposite internal
carotid artery and external meningeal feeders

A

A. The size and location of the tumor relative to the optic
nerve
B. The direction of venous drainage and rate of blood flow
through the shunt

C. The extent of the inflammatory reaction adjacent to
the cavernous sinus
D. The extent of the inflammatory reaction adjacent to
the superior orbital fissure
E. The extent of collateral flow from the opposite internal
carotid artery and external meningeal feeders

Carotid-cavernous fistulas (CCFs) can be
divided into posttraumatic and spontaneous types. They are
direct shunts between the ICA or ECA and cavernous sinus
and usually occur after trauma or spontaneous aneurysmal
rupture. Traumatic CCFs often present in a delayed fashion ;
like spontaneous fistulas, they often present with retro-orbital
199
200 Intensive Neurosurgery Board Review
pain, chemosis, pulsatile proptosis, ocular or cranial bruit,
decreased visual acuity, diplopia, and rarely epistaxis and
subarachnoid hemorrhage. The symptoms depend on the
direction of venous flow and quantity of blood flow through
the fistula. There are four types of CCFs: type A is a direct,
high-flow shunt between the ICA and cavernous sinus (as
in this case), and types B to D are low-flow shunts between
the cavernous sinus and meningeal branches of the internal
carotid artery, external carotid artery, or both, respectively. Approximately 50% of low-flow fistulas spontaneously
thrombose without treatment. The main treatment option
has traditionally included transarterial balloon embolization
through the ICA for type A fistulas, although accessing the
fistula transvenously (Le., inferior petrosal sinus) is also
commonly performed, especially for indirect types B to D.
A direct surgical approach is indicated if transarterial or
transvenous approaches fail. Radiosurgery has been proposed as an option for some of the low-flow fistulas, although
it would not be the best strategy for the high-flow symptomatic fistula seen in this patient. Figure A depicts nearly
complete capture of the blood from the internal carotid
artery, and fistulous drainage primarily from the superior
ophthalmic and superior petrosal veins. Figure B depicts a
later venous run with superior petrosal vein drainage into the
transverse-sigmoid sinus junction as well as some venous
drainage into the superior sagittal sinus (Kaye and Blacl~,
pp. 1132; Greenberg, pp. 811- 812; Youmans, pp. 2341-
2352; Will~ins, pp. 2529- 2535).

How well did you know this?
1
Not at all
2
3
4
5
Perfectly
8
Q

A 28-year-old male was involved in a motorcycle
accident. About 1 week after being discharged from the
hospital he began experiencing fevers, severe retroorbital
headaches, diplopia, and left eye proptosis, which prompted
a visit to the emergency department. A computed tomography
(CT) scan of the brain showed a ‘ resolving 2- by 3-cm left
frontal contusion underlying a minimally displaced frontal
bone fracture, which was sustained at the time of initial
injury. His erythrocyte sedimentation rate (ESR) and Creactive protein (CRP) were mildly elevated. The angiogram
is depicted below

What should be the initial treatment of choice for this
patient?

A. Six weeks of antibiotics followed by repeat angiography
B. Glue embolization of major arterial feeders followed by
tumor resection
C. Carotid artery sacrifice
D. Transarterial detachable balloon embolization
E. Heparin infusion

A

A. Six weeks of antibiotics followed by repeat angiography
B. Glue embolization of major arterial feeders followed by
tumor resection
C. Carotid artery sacrifice
**D. Transarterial detachable balloon embolization **
E. Heparin infusion

Carotid-cavernous fistulas (CCFs) can be
divided into posttraumatic and spontaneous types. They are
direct shunts between the ICA or ECA and cavernous sinus
and usually occur after trauma or spontaneous aneurysmal
rupture. Traumatic CCFs often present in a delayed fashion ;
like spontaneous fistulas, they often present with retro-orbital
199
200 Intensive Neurosurgery Board Review
pain, chemosis, pulsatile proptosis, ocular or cranial bruit,
decreased visual acuity, diplopia, and rarely epistaxis and
subarachnoid hemorrhage. The symptoms depend on the
direction of venous flow and quantity of blood flow through
the fistula. There are four types of CCFs: type A is a direct,
high-flow shunt between the ICA and cavernous sinus (as
in this case), and types B to D are low-flow shunts between
the cavernous sinus and meningeal branches of the internal
carotid artery, external carotid artery, or both, respectively. Approximately 50% of low-flow fistulas spontaneously
thrombose without treatment. The main treatment option
has traditionally included transarterial balloon embolization
through the ICA for type A fistulas, although accessing the
fistula transvenously (Le., inferior petrosal sinus) is also
commonly performed, especially for indirect types B to D.
A direct surgical approach is indicated if transarterial or
transvenous approaches fail. Radiosurgery has been proposed as an option for some of the low-flow fistulas, although
it would not be the best strategy for the high-flow symptomatic fistula seen in this patient. Figure A depicts nearly
complete capture of the blood from the internal carotid
artery, and fistulous drainage primarily from the superior
ophthalmic and superior petrosal veins. Figure B depicts a
later venous run with superior petrosal vein drainage into the
transverse-sigmoid sinus junction as well as some venous
drainage into the superior sagittal sinus (Kaye and Blacl~,
pp. 1132; Greenberg, pp. 811- 812; Youmans, pp. 2341-
2352; Will~ins, pp. 2529- 2535).

How well did you know this?
1
Not at all
2
3
4
5
Perfectly
9
Q

A 28-year-old male was involved in a motorcycle
accident. About 1 week after being discharged from the
hospital he began experiencing fevers, severe retroorbital
headaches, diplopia, and left eye proptosis, which prompted
a visit to the emergency department. A computed tomography
(CT) scan of the brain showed a ‘ resolving 2- by 3-cm left
frontal contusion underlying a minimally displaced frontal
bone fracture, which was sustained at the time of initial
injury. His erythrocyte sedimentation rate (ESR) and Creactive protein (CRP) were mildly elevated. The angiogram
is depicted below

If the desired treatment strategy fails, what would be
another potential treatment option?
1. Surgical debridement of the infection
2. Direct surgical packing of the cavernous sinus with
either Gelfoam, Surgicel, platinum coils, or strands of
cotton
3. Preoperative glue embolization of arterial feeders
followed by tumor resection
4. Endovascular procedure for internal carotid artery
sacrifice

A. 1, 2, and3 are correct
B. 1 and3 are correct
C. 2 and4 are correct
D. Only 4 is correct
E. All of the above

A

A. 1, 2, and3 are correct
B. 1 and3 are correct
**C. 2 and4 are correct **
D. Only 4 is correct
E. All of the above

Carotid-cavernous fistulas (CCFs) can be
divided into posttraumatic and spontaneous types. They are
direct shunts between the ICA or ECA and cavernous sinus
and usually occur after trauma or spontaneous aneurysmal
rupture. Traumatic CCFs often present in a delayed fashion ;
like spontaneous fistulas, they often present with retro-orbital
199
200 Intensive Neurosurgery Board Review
pain, chemosis, pulsatile proptosis, ocular or cranial bruit,
decreased visual acuity, diplopia, and rarely epistaxis and
subarachnoid hemorrhage. The symptoms depend on the
direction of venous flow and quantity of blood flow through
the fistula. There are four types of CCFs: type A is a direct,
high-flow shunt between the ICA and cavernous sinus (as
in this case), and types B to D are low-flow shunts between
the cavernous sinus and meningeal branches of the internal
carotid artery, external carotid artery, or both, respectively. Approximately 50% of low-flow fistulas spontaneously
thrombose without treatment. The main treatment option
has traditionally included transarterial balloon embolization
through the ICA for type A fistulas, although accessing the
fistula transvenously (Le., inferior petrosal sinus) is also
commonly performed, especially for indirect types B to D.
A direct surgical approach is indicated if transarterial or
transvenous approaches fail. Radiosurgery has been proposed as an option for some of the low-flow fistulas, although
it would not be the best strategy for the high-flow symptomatic fistula seen in this patient. Figure A depicts nearly
complete capture of the blood from the internal carotid
artery, and fistulous drainage primarily from the superior
ophthalmic and superior petrosal veins. Figure B depicts a
later venous run with superior petrosal vein drainage into the
transverse-sigmoid sinus junction as well as some venous
drainage into the superior sagittal sinus (Kaye and Blacl~,
pp. 1132; Greenberg, pp. 811- 812; Youmans, pp. 2341-
2352; Will~ins, pp. 2529- 2535).

How well did you know this?
1
Not at all
2
3
4
5
Perfectly
10
Q

‘Vhat finding in the pathologic process depicted by
the angiogram below (Figure 6.10Q) would mandate urgent
treatment?

A. Retrograde cortical venous drainage
B. J’vlultiple meningeal artery feeders
C. Dual internal and external carotid artery supply
D. Embolic stroke
E. Venous sinus occlusion

A

**A. Retrograde cortical venous drainage **
B. J’vlultiple meningeal artery feeders
C. Dual internal and external carotid artery supply
D. Embolic stroke
E. Venous sinus occlusion

The natural history of DAVF is variable and includes
spontaneous resolution, recruitment of meningeal arterial
feeders, and the development of intracranial hypertension.
DAVF can present with pulsatile tinnitus, visual symptoms,
papilledema , hydrocephalus, and intracranial hemorrhage.
The presence of retrograde cortical venous drainage indicates
the potential for intracranial hemorrhage and mandates
urgent treatment of the DAVF. Intracranial hemorrhage from
a DA VF in the absence of retrograde cortical venous drainage
has not been reported. Hemorrhage from a DA VF is associated with a high morbidity and mortality (approximately
30%). Ectatic dilation or venous occlusion of the involved
sinus, multiple or dual ICiVECA arterial feeders, or embolic
stroke, in the absence of retrograde cortical venous drainage
has not been reported to increase hemorrhage rates of
DAVFs (Kaye and Blaci<, pp. 1125-1135; Greenberg, p. 811;
Youmans, pp. 2171- 2173; Will~ins, pp. 2523-2527).

How well did you know this?
1
Not at all
2
3
4
5
Perfectly
11
Q

A 67-year-old male with a history of diabetes
mellitus and hypertension presents to the emergency department with right arm wealmess and numbness. He is found to
have> 90% stenosis of the left internal carotid artery and
restricted MR diffusion in portions of the brain supplied by
the left middle cerebral artery. He elects to proceed with
surgery for his carotid stenosis but is found to have a highriding carotid artery bifurcation.

Surgical maneuvers that may increase surgical exposure
of a high-riding carotid artery bifurcation during carotid
endarterectomy include all of the following EXCEPT?

A. Medial mobilization of the ansa cervicalis
B. Dividing the posterior belly of the digastric muscle
C. l’vIandibular osteotomy or disarticulation of the mandible
at the temporomandibular joint
D. Judicious cautery and ligation of select vessels (occipital
artery, common facial vein) hindering exposure
E. Transverse sectioning of the clavicular head of the
sternocleidomastoid muscle at the level of the hyoid
bone for better visualization of the carotid artery
lateral to the jugular vein

A

A. Medial mobilization of the ansa cervicalis
B. Dividing the posterior belly of the digastric muscle
C. l’vIandibular osteotomy or disarticulation of the mandible
at the temporomandibular joint
D. Judicious cautery and ligation of select vessels (occipital
artery, common facial vein) hindering exposure
**E. Transverse sectioning of the clavicular head of the
sternocleidomastoid muscle at the level of the hyoid
bone for better visualization of the carotid artery
lateral to the jugular vein
**

Attempts to gain additional
exposure for a high-riding carotid artery bifurcation include
mobilization of the ansa cervicalis, sectioning the posterior
belly of the digastric muscle , cautery and ligation of the
occipital artery, and mandibular osteotomy or disarticulation of the jaw at the temporomandibular joint. This type
of exposure places the hypoglossal nerve at particular risk,
although segments of cranial nerves VII, DC, X, and Xl can also be injured during carotid endarterectomy (CEA).
Patients who become hypotensive and bradycardic during
surgery often do so as a result of manipulation of the nerve of
Hering near the carotid bulb. This is not uncommon with
CEA and can often be addressed with lidocaine infusion adjacent to the carotid bulb. Placing the clamps on the internal
carotid artery first, followed by the common and then the
external carotid artery often ensures that the clot will pass
through the external carotid artery instead of the internal
carotid artery. The order for clamp removal should be just
the opposite, as this should again ensure that any accumulated blood clot will be more likely to pass through the
external rather than internal carotid circulation. It is not
uncommon during CEA to have some backbleeding into the
surgical field by the ascending pharyngeal artery after clamp
placement on the major vessels. If the extent of bleeding is
severe and hinders the operation, identification, clamping
(aneurysm clip), or. ligation of this vessel may drastically
improve visibility. A patient who awakens with a major
neurologic deficit is likely to have suffered thrombosis at the
arteriotomy site, which usually warrants immediate attention (surgical exploration) rather than time-consuming
diagnostic studies, as some case reports describe a significant neurologic improvement if flow is re-established
within 45 minutes. For later-onset deficits, workup (Le. ,
CT, angiogram) may be indicated. CT may help to identify
hemorrhage and an angiogram may reveal whether the ICA
is occluded or if the deficit is from another cause (emboli)
that would not necessarily require surgical re-exploration
(Kaye and Blacl~, pp. 1179- 1187; Greenberg, pp. 837-841;
Youmans, pp. 1631-1645; Will~ins, pp. 2113- 2114).

How well did you know this?
1
Not at all
2
3
4
5
Perfectly
12
Q

A 67-year-old male with a history of diabetes
mellitus and hypertension presents to the emergency department with right arm wealmess and numbness. He is found to
have> 90% stenosis of the left internal carotid artery and
restricted MR diffusion in portions of the brain supplied by
the left middle cerebral artery. He elects to proceed with
surgery for his carotid stenosis but is found to have a highriding carotid artery bifurcation.

What cranial nerve is at most risk of injury when exposing a high-riding carotid artery bifurcation?

A. VII
B. IX
C. X
D. XI
E. XII

A

A. VII
B. IX
C. X
D. XI
E. XII

Attempts to gain additional
exposure for a high-riding carotid artery bifurcation include
mobilization of the ansa cervicalis, sectioning the posterior
belly of the digastric muscle , cautery and ligation of the
occipital artery, and mandibular osteotomy or disarticulation of the jaw at the temporomandibular joint. This type
of exposure places the hypoglossal nerve at particular risk,
although segments of cranial nerves VII, DC, X, and Xl can also be injured during carotid endarterectomy (CEA).
Patients who become hypotensive and bradycardic during
surgery often do so as a result of manipulation of the nerve of
Hering near the carotid bulb. This is not uncommon with
CEA and can often be addressed with lidocaine infusion adjacent to the carotid bulb. Placing the clamps on the internal
carotid artery first, followed by the common and then the
external carotid artery often ensures that the clot will pass
through the external carotid artery instead of the internal
carotid artery. The order for clamp removal should be just
the opposite, as this should again ensure that any accumulated blood clot will be more likely to pass through the
external rather than internal carotid circulation. It is not
uncommon during CEA to have some backbleeding into the
surgical field by the ascending pharyngeal artery after clamp
placement on the major vessels. If the extent of bleeding is
severe and hinders the operation, identification, clamping
(aneurysm clip), or. ligation of this vessel may drastically
improve visibility. A patient who awakens with a major
neurologic deficit is likely to have suffered thrombosis at the
arteriotomy site, which usually warrants immediate attention (surgical exploration) rather than time-consuming
diagnostic studies, as some case reports describe a significant neurologic improvement if flow is re-established
within 45 minutes. For later-onset deficits, workup (Le. ,
CT, angiogram) may be indicated. CT may help to identify
hemorrhage and an angiogram may reveal whether the ICA
is occluded or if the deficit is from another cause (emboli)
that would not necessarily require surgical re-exploration
(Kaye and Blacl~, pp. 1179- 1187; Greenberg, pp. 837-841;
Youmans, pp. 1631-1645; Will~ins, pp. 2113- 2114).

How well did you know this?
1
Not at all
2
3
4
5
Perfectly
13
Q

A 67-year-old male with a history of diabetes
mellitus and hypertension presents to the emergency department with right arm wealmess and numbness. He is found to
have> 90% stenosis of the left internal carotid artery and
restricted MR diffusion in portions of the brain supplied by
the left middle cerebral artery. He elects to proceed with
surgery for his carotid stenosis but is found to have a highriding carotid artery bifurcation.

What is the order of clamp placement on the arteries
during carotid endarterectomy?

A. External, internal, common
B. Internal, common, external
C. External, common, internal
D. Common, external, internal
E. Common, internal, external

A

A. External, internal, common
**B. Internal, common, external **
C. External, common, internal
D. Common, external, internal
E. Common, internal, external

Attempts to gain additional
exposure for a high-riding carotid artery bifurcation include
mobilization of the ansa cervicalis, sectioning the posterior
belly of the digastric muscle , cautery and ligation of the
occipital artery, and mandibular osteotomy or disarticulation of the jaw at the temporomandibular joint. This type
of exposure places the hypoglossal nerve at particular risk,
although segments of cranial nerves VII, DC, X, and Xl can also be injured during carotid endarterectomy (CEA).
Patients who become hypotensive and bradycardic during
surgery often do so as a result of manipulation of the nerve of
Hering near the carotid bulb. This is not uncommon with
CEA and can often be addressed with lidocaine infusion adjacent to the carotid bulb. Placing the clamps on the internal
carotid artery first, followed by the common and then the
external carotid artery often ensures that the clot will pass
through the external carotid artery instead of the internal
carotid artery. The order for clamp removal should be just
the opposite, as this should again ensure that any accumulated blood clot will be more likely to pass through the
external rather than internal carotid circulation. It is not
uncommon during CEA to have some backbleeding into the
surgical field by the ascending pharyngeal artery after clamp
placement on the major vessels. If the extent of bleeding is
severe and hinders the operation, identification, clamping
(aneurysm clip), or. ligation of this vessel may drastically
improve visibility. A patient who awakens with a major
neurologic deficit is likely to have suffered thrombosis at the
arteriotomy site, which usually warrants immediate attention (surgical exploration) rather than time-consuming
diagnostic studies, as some case reports describe a significant neurologic improvement if flow is re-established
within 45 minutes. For later-onset deficits, workup (Le. ,
CT, angiogram) may be indicated. CT may help to identify
hemorrhage and an angiogram may reveal whether the ICA
is occluded or if the deficit is from another cause (emboli)
that would not necessarily require surgical re-exploration
(Kaye and Blacl~, pp. 1179- 1187; Greenberg, pp. 837-841;
Youmans, pp. 1631-1645; Will~ins, pp. 2113- 2114).

How well did you know this?
1
Not at all
2
3
4
5
Perfectly
14
Q

A 67-year-old male with a history of diabetes
mellitus and hypertension presents to the emergency department with right arm wealmess and numbness. He is found to
have> 90% stenosis of the left internal carotid artery and
restricted MR diffusion in portions of the brain supplied by
the left middle cerebral artery. He elects to proceed with
surgery for his carotid stenosis but is found to have a highriding carotid artery bifurcation.

After clamp placement and arteriotomy, the surgeon
notices continued bleeding from the back wall of the carotid
artery, which severely hinders visualization during the
surgical procedure. What is the most likely reason for the
continued bleeding?
A. Incomplete clamping of the common carotid artery
B. Backbleeding from the superficial temporal artery
C. Backbleeding from the ascending pharyngeal artery
D. Venous bleeding from the adventitia of the internal
carotid artery
E. Clotting abnormality from heparin inhlsion

A

A. Incomplete clamping of the common carotid artery
B. Backbleeding from the superficial temporal artery
**C. Backbleeding from the ascending pharyngeal artery **
D. Venous bleeding from the adventitia of the internal
carotid artery
E. Clotting abnormality from heparin inhlsion

Attempts to gain additional
exposure for a high-riding carotid artery bifurcation include
mobilization of the ansa cervicalis, sectioning the posterior
belly of the digastric muscle , cautery and ligation of the
occipital artery, and mandibular osteotomy or disarticulation of the jaw at the temporomandibular joint. This type
of exposure places the hypoglossal nerve at particular risk,
although segments of cranial nerves VII, DC, X, and Xl can also be injured during carotid endarterectomy (CEA).
Patients who become hypotensive and bradycardic during
surgery often do so as a result of manipulation of the nerve of
Hering near the carotid bulb. This is not uncommon with
CEA and can often be addressed with lidocaine infusion adjacent to the carotid bulb. Placing the clamps on the internal
carotid artery first, followed by the common and then the
external carotid artery often ensures that the clot will pass
through the external carotid artery instead of the internal
carotid artery. The order for clamp removal should be just
the opposite, as this should again ensure that any accumulated blood clot will be more likely to pass through the
external rather than internal carotid circulation. It is not
uncommon during CEA to have some backbleeding into the
surgical field by the ascending pharyngeal artery after clamp
placement on the major vessels. If the extent of bleeding is
severe and hinders the operation, identification, clamping
(aneurysm clip), or. ligation of this vessel may drastically
improve visibility. A patient who awakens with a major
neurologic deficit is likely to have suffered thrombosis at the
arteriotomy site, which usually warrants immediate attention (surgical exploration) rather than time-consuming
diagnostic studies, as some case reports describe a significant neurologic improvement if flow is re-established
within 45 minutes. For later-onset deficits, workup (Le. ,
CT, angiogram) may be indicated. CT may help to identify
hemorrhage and an angiogram may reveal whether the ICA
is occluded or if the deficit is from another cause (emboli)
that would not necessarily require surgical re-exploration
(Kaye and Blacl~, pp. 1179- 1187; Greenberg, pp. 837-841;
Youmans, pp. 1631-1645; Will~ins, pp. 2113- 2114).

How well did you know this?
1
Not at all
2
3
4
5
Perfectly
15
Q

A 67-year-old male with a history of diabetes
mellitus and hypertension presents to the emergency department with right arm wealmess and numbness. He is found to
have> 90% stenosis of the left internal carotid artery and
restricted MR diffusion in portions of the brain supplied by
the left middle cerebral artery. He elects to proceed with
surgery for his carotid stenosis but is found to have a highriding carotid artery bifurcation.

During surgical dissection adjacent to the carotid
artery, the anesthesiologist notices that the patient becomes
hypotensive and bradycardic. The next course of management should include

A. Obtain an immediate arterial blood gas (ABG) to
determine if the patient is suffering from a pulmonary
embolus
B. Check cardiac enzymes, as the patient is likely suffering from an anterior myocardial wall infarction
C. The nerve to the carotid sinus (nerve of Hering) should
be anesthetized with 0.5 mL of 2% lidocaine
D. Begin dobutamine , check central venous pressures,
and obtain a lactate level, as the patient is likely to be
volume-depleted
E. 100 IV/kg of heparin should be infused intravenously
to prevent further emboli

A

A. Obtain an immediate arterial blood gas (ABG) to
determine if the patient is suffering from a pulmonary
embolus
B. Check cardiac enzymes, as the patient is likely suffering from an anterior myocardial wall infarction
**C. The nerve to the carotid sinus (nerve of Hering) should
be anesthetized with 0.5 mL of 2% lidocaine **
D. Begin dobutamine , check central venous pressures,
and obtain a lactate level, as the patient is likely to be
volume-depleted
E. 100 IV/kg of heparin should be infused intravenously
to prevent further emboli

Attempts to gain additional
exposure for a high-riding carotid artery bifurcation include
mobilization of the ansa cervicalis, sectioning the posterior
belly of the digastric muscle , cautery and ligation of the
occipital artery, and mandibular osteotomy or disarticulation of the jaw at the temporomandibular joint. This type
of exposure places the hypoglossal nerve at particular risk,
although segments of cranial nerves VII, DC, X, and Xl can also be injured during carotid endarterectomy (CEA).
Patients who become hypotensive and bradycardic during
surgery often do so as a result of manipulation of the nerve of
Hering near the carotid bulb. This is not uncommon with
CEA and can often be addressed with lidocaine infusion adjacent to the carotid bulb. Placing the clamps on the internal
carotid artery first, followed by the common and then the
external carotid artery often ensures that the clot will pass
through the external carotid artery instead of the internal
carotid artery. The order for clamp removal should be just
the opposite, as this should again ensure that any accumulated blood clot will be more likely to pass through the
external rather than internal carotid circulation. It is not
uncommon during CEA to have some backbleeding into the
surgical field by the ascending pharyngeal artery after clamp
placement on the major vessels. If the extent of bleeding is
severe and hinders the operation, identification, clamping
(aneurysm clip), or. ligation of this vessel may drastically
improve visibility. A patient who awakens with a major
neurologic deficit is likely to have suffered thrombosis at the
arteriotomy site, which usually warrants immediate attention (surgical exploration) rather than time-consuming
diagnostic studies, as some case reports describe a significant neurologic improvement if flow is re-established
within 45 minutes. For later-onset deficits, workup (Le. ,
CT, angiogram) may be indicated. CT may help to identify
hemorrhage and an angiogram may reveal whether the ICA
is occluded or if the deficit is from another cause (emboli)
that would not necessarily require surgical re-exploration
(Kaye and Blacl~, pp. 1179- 1187; Greenberg, pp. 837-841;
Youmans, pp. 1631-1645; Will~ins, pp. 2113- 2114).

How well did you know this?
1
Not at all
2
3
4
5
Perfectly
16
Q

A 67-year-old male with a history of diabetes
mellitus and hypertension presents to the emergency department with right arm wealmess and numbness. He is found to
have> 90% stenosis of the left internal carotid artery and
restricted MR diffusion in portions of the brain supplied by
the left middle cerebral artery. He elects to proceed with
surgery for his carotid stenosis but is found to have a highriding carotid artery bifurcation.

Postoperatively, the patient awoke with right-sided
hemiplegia and lethargy. The next logical course of management should include

A. Immediate CT angiography to assess the patency of the
right carotid artery
B. Immediate selective angiography of the right carotid
artery
C. Antiplatelet therapy for 1 week, followed by repeat
angiography
D. Stent placement across the arteriotomy site to reinforce the closure
E. Immediate surgical reexploration for thrombectomy

A

A. Immediate CT angiography to assess the patency of the
right carotid artery
B. Immediate selective angiography of the right carotid
artery
C. Antiplatelet therapy for 1 week, followed by repeat
angiography
D. Stent placement across the arteriotomy site to reinforce the closure
E. Immediate surgical reexploration for thrombectomy

Attempts to gain additional
exposure for a high-riding carotid artery bifurcation include
mobilization of the ansa cervicalis, sectioning the posterior
belly of the digastric muscle , cautery and ligation of the
occipital artery, and mandibular osteotomy or disarticulation of the jaw at the temporomandibular joint. This type
of exposure places the hypoglossal nerve at particular risk,
although segments of cranial nerves VII, DC, X, and Xl can also be injured during carotid endarterectomy (CEA).
Patients who become hypotensive and bradycardic during
surgery often do so as a result of manipulation of the nerve of
Hering near the carotid bulb. This is not uncommon with
CEA and can often be addressed with lidocaine infusion adjacent to the carotid bulb. Placing the clamps on the internal
carotid artery first, followed by the common and then the
external carotid artery often ensures that the clot will pass
through the external carotid artery instead of the internal
carotid artery. The order for clamp removal should be just
the opposite, as this should again ensure that any accumulated blood clot will be more likely to pass through the
external rather than internal carotid circulation. It is not
uncommon during CEA to have some backbleeding into the
surgical field by the ascending pharyngeal artery after clamp
placement on the major vessels. If the extent of bleeding is
severe and hinders the operation, identification, clamping
(aneurysm clip), or. ligation of this vessel may drastically
improve visibility. A patient who awakens with a major
neurologic deficit is likely to have suffered thrombosis at the
arteriotomy site, which usually warrants immediate attention (surgical exploration) rather than time-consuming
diagnostic studies, as some case reports describe a significant neurologic improvement if flow is re-established
within 45 minutes. For later-onset deficits, workup (Le. ,
CT, angiogram) may be indicated. CT may help to identify
hemorrhage and an angiogram may reveal whether the ICA
is occluded or if the deficit is from another cause (emboli)
that would not necessarily require surgical re-exploration
(Kaye and Blacl~, pp. 1179- 1187; Greenberg, pp. 837-841;
Youmans, pp. 1631-1645; Will~ins, pp. 2113- 2114).

How well did you know this?
1
Not at all
2
3
4
5
Perfectly
17
Q

A lS-year-old female undergoes uncomplicated resection of
the lesion depicted below (Figure 6.17-6.18Q). Four days
later she develops lethargy, fever, meningismus, and photophobia. A cerebrospinal fluid (CSF) sample reveals a protein
level of 86 mg/dL (reference range, 12 to 60 mg/dL), a glucose level of 61 mg/dL (reference range , 40 to 70 mg/dL),
16 red blood cells/mL, and 126 white blood celis/mL with a
differential of 11% neutrophils, 82% lymphocytes, and 7%
histiocytes. Gram stain and culture of CSF were sterile and
remained so for the presence of organisms.

Wha t is the most likely diagnosis ?

A. Bacterial meningitis
B. Aseptic meningitis
C. Hydrocephalus
D. Postmeningitis syndrome
E. Viral encephalitis

A

A. Bacterial meningitis
**B. Aseptic meningitis **
C. Hydrocephalus
D. Postmeningitis syndrome
E. Viral encephalitis

AseptiC meningitis (AM) is a well-recognized
complication after posterior fossa surgery but is typically
self-limited and requires no treatment. It has generally been
attributed to one or more irritants released into the subarachnoid space during surgery, including blood breakdown
products, tumor, muscle , and brain. Lowering of intracranial
pressure with lumbar puncture and dexamethasone is the
mainstay of treatment in certain patients with continued ,
problems. Bacterial meningitis and postmeningitic syndrome are unlikely, considering that an organism was not
isolated from the CSF, although this is not always the case.
lVloreover, the CSF profile was more consistent with aseptic
meningi tis than bacterial meningitis. Hydrocephalus is
unlikely, since fever, meningismus, and photophobia rarely
accompany this diagnosis, and encephalitis would be very
uncommon in this situation (Carmel et ai., pp. 276-280;
Youmans, pp. 3645, 3659; Kaye and Blacl~, p. 868; Will~ins,
p.3965).

How well did you know this?
1
Not at all
2
3
4
5
Perfectly
18
Q

A lS-year-old female undergoes uncomplicated resection of
the lesion depicted below (Figure 6.17-6.18Q). Four days
later she develops lethargy, fever, meningismus, and photophobia. A cerebrospinal fluid (CSF) sample reveals a protein
level of 86 mg/dL (reference range, 12 to 60 mg/dL), a glucose level of 61 mg/dL (reference range , 40 to 70 mg/dL),
16 red blood cells/mL, and 126 white blood celis/mL with a
differential of 11% neutrophils, 82% lymphocytes, and 7%
histiocytes. Gram stain and culture of CSF were sterile and
remained so for the presence of organisms.

What is the natural history of this problem?
A. Requires a 10-day course of antibiotics despite negative
cultures to cover for slow-growing bacterial species
B. Patients frequently require steroid therapy followed
by repeat lumbar punctures
C. Usually self-limited and requires no treatment
D. Patients show drastic improvement with shunting
E. Usually favorable once any synthetic material placed
during surgery (e.g., dural graft) is removed

A

A. Requires a 10-day course of antibiotics despite negative
cultures to cover for slow-growing bacterial species
B. Patients frequently require steroid therapy followed
by repeat lumbar punctures
C. Usually self-limited and requires no treatment
D. Patients show drastic improvement with shunting
E. Usually favorable once any synthetic material placed
during surgery (e.g., dural graft) is removed

AseptiC meningitis (AM) is a well-recognized
complication after posterior fossa surgery but is typically
self-limited and requires no treatment. It has generally been
attributed to one or more irritants released into the subarachnoid space during surgery, including blood breakdown
products, tumor, muscle , and brain. Lowering of intracranial
pressure with lumbar puncture and dexamethasone is the
mainstay of treatment in certain patients with continued ,
problems. Bacterial meningitis and postmeningitic syndrome are unlikely, considering that an organism was not
isolated from the CSF, although this is not always the case.
lVloreover, the CSF profile was more consistent with aseptic
meningi tis than bacterial meningitis. Hydrocephalus is
unlikely, since fever, meningismus, and photophobia rarely
accompany this diagnosis, and encephalitis would be very
uncommon in this situation (Carmel et ai., pp. 276-280;
Youmans, pp. 3645, 3659; Kaye and Blacl~, p. 868; Will~ins,
p.3965).

How well did you know this?
1
Not at all
2
3
4
5
Perfectly
19
Q

A 62-year-old female undergoes microvascular decompression for hemifacial spasm. Postoperatively, she has complete ipsilateral deafness but no other neurologic deficits.
The most likely cause of this deficit was injury to one of the
blood vessels that originated from which artery?
A. Posterior cerebral artery (PCA)
B. Superior cerebellar artery (SCA)
C. Anterior inferior cerebellar artery (AICA)
D. Posterior inferior cerebellar artery (PICA)
E. Vertebral artery

A

A. Posterior cerebral artery (PCA)
B. Superior cerebellar artery (SCA)
**C. Anterior inferior cerebellar artery (AICA) **
D. Posterior inferior cerebellar artery (PICA)
E. Vertebral artery

Complications of microvascular decompression for
hemifacial spasm include CSF leak, facial weakness, facial
anesthesia, corneal anesthesia, intracranial hemorrhage,
and infarction. Complete ipsilateral deafness is associated
with disruption or coagulation of the labyrinthine artery,
which is most commonly a branch of either the AlCA
(45%), SCA (25%), or basilar artery (16%) (Kaye and Black,
pp. 1652-1653; Osborne DN, p. 186; Greenberg, pp. 358-
360; Youmans, pp. 3013-3014; Wil~<ins, pp. 3227-3233).

How well did you know this?
1
Not at all
2
3
4
5
Perfectly
20
Q

A 14-year-old girl with progressive loss of vision in her
right eye was recently diagnosed with a 2.0- by 3.5-cm right
optic nerve glioma extending to the optic chiasm. During
surgery, the portion of the tumor on the optic nerve was
successfully resected, but the tumor adjacent to the optic
chiasm was left behind. What is the maximal dose of singlefraction radiosurgery that can safely be employed to the
optic chiasm?
A. 4 to 7 Gy
B. 9 to 10 Gy
C. 11 to 13 Gy
D. 14 to 16 Gy
E. 21 Gy

A

A. 4 to 7 Gy
**B. 9 to 10 Gy **
C. 11 to 13 Gy
D. 14 to 16 Gy
E. 21 Gy

The maximal safe dose of Single-shot radiosurgelY
that the optic chiasm can tolerate is approximately 9 to
10 Gy (Alexander, p. 171).

How well did you know this?
1
Not at all
2
3
4
5
Perfectly
21
Q

A surgeon decides to utilize an infratentorialsupra cerebellar corridor to approach a pineal region mass.
What blood vessel is frequently cauterized and divided
for better exposure of the posterior surface of the tumor
during this approach?

A. Vein of Galen
B. Ipsilateral basal vein of Rosenthal
C. Posterior cerebral artery (PCA)
D. Precentral cerebellar vein
E. Superior petrosal sinus

A

A. Vein of Galen
B. Ipsilateral basal vein of Rosenthal
C. Posterior cerebral artery (PCA)
**D. Precentral cerebellar vein **
E. Superior petrosal sinus

Cauterizing and dividing the precentral cerebellar
vein will often expose the posterior surface of pineal region
tumors. The veins of Galen and Rosenthal should be preserved during this operation, as well as the vennian vein,
which often can be spared in this approach. The choroidal
arteries may supply feeders to the tumor but rarely need to
be cauterized and ligated for adequate tumor resection (Kaye
and Black, pp. 815-824; Youmans; pp. 1017- 1021, Will<ins,
p.l029).

How well did you know this?
1
Not at all
2
3
4
5
Perfectly
22
Q

During translabyrinthine exposure for acoustic neuroma
resection, surgeons find themselves exposing Trautmann’s
triangle. All of the following structures delineate this area
EXCEPT?

A. A triangular patch of dura on the posterior aspect of
the temporal bone facing the cei’ebellopontine angle
B. The sigmoid sinus laterally
C. The superior petrosal sinus above
D. The jugular bulb below
E. The foramen magnum medially

A

A. A triangular patch of dura on the posterior aspect of
the temporal bone facing the cei’ebellopontine angle
B. The sigmoid sinus laterally
C. The superior petrosal sinus above
D. The jugular bulb below
**E. The foramen magnum medially **

There are two goals of the translabyrinthine approach
for acoustic neuroma resection that may help achieve m~’dmal tumor resection. The first is to remove enough bone
to identify the nerves lateral to the tumor as they course
through the lAC, and the second is to expose the dura of the
posterior aspect of the temporal bone that faces the cerebellopontine angle (CPA). This triangular patch of dura facing
the CPA is called Trautmann’s triangle and extends from
the sigmoid sinus laterally, the superior petrosal sinus
above, and the jugular bulb below. The foramen magnum
is not included in Trautmann’s triangle (Kaye and Black,
pp. 851-860; Youmans, pp. 1155- 1156; Wilkins, pp. 1067-
1071).

How well did you know this?
1
Not at all
2
3
4
5
Perfectly
23
Q

One of the earliest procedures performed for Parkinson’s
disease was ligation of what blood vessel ?
A. Anterior choroidal artery
B. Medial posterior choroidal artery
C. Recurrent artery of Heubner
D. Tentorial artery of Bernasconi and Cassarini
E. Medial lenticulostriate artery

A

A. Anterior choroidal artery
B. Medial posterior choroidal artery
C. Recurrent artery of Heubner
D. Tentorial artery of Bernasconi and Cassarini
E. Medial lenticulostriate artery

Neurosurgical therapies for Parkinson’s disease (PD)
have been utilized in patients with progressive disease
despite maximal medical therapy. An early procedure performed for PD was ligation of the anterior choroidal artery,
with subsequent infarction of the pallidum. Due to the variable distribution of this vessel outside the confines of the pallidum, results were too unpredictable and this procedure lost
favor. In the 1950s, anterodorsal pallidotomy became an
accepted procedure, but the long-term benefits were mostly
for rigidity, while tremor and dyskinesia did not improve.
Subsequently, the ventrolateral thalamus became the preferred target for lesioning, but this procedure also lost
favor, as patients were often still left with bradykinesia
and/or rigidity. Moreover, this procedure reduced tremor
only in the contralateral half of the body, and bilateral thalamotomies were not recommended due to an unacceptably
high risk of postoperative dysarthria and gait disturbances.
Thalamotomy procedures fell off dramatically in the late
1960s with the introduction of L-DOPA.
J\·Iore recently, dramatic and beneficial effects of both
subthalamic nucleus (STN) and globus pallidus interna (Gpi) deep brain stimulation (DBS) have been consistently
observed. Both interventions appear to result in significant
improvements in both motor fluctuations and dyskinesias.
The DBS study group, in a large multicenter study, reported
that on time without dyskinesia during the walling hours
increased from 25 to 30% at baseline to 65 to 75% 6 months
postoperatively. In a complementary fashion , these procedures also markedly decreased off time and on time without
dyskinesia. Although some preliminary studies suggest STN
DBS may be a superior intervention, no large randomized
controlled trial comparing STN and Gpi DBS has been conducted to compare the efficacy of these treatments. The most
consistent finding has been the reduction in antiparkinson
medication following STN DBS compared to Gpi DBS.
(Greenberg, p. 751; Tarsy, p. 191).

How well did you know this?
1
Not at all
2
3
4
5
Perfectly
24
Q

Vagal nerve stimulation is reserved for select patients
with epilepsy. Why is it performed on the left side?
A. To avoid injuring the recurrent laryngeal nerve, which
follows a more torturous route on the right
B. To avoid damage to the dominant superior laryngeal
nerve on the right
C. To avoid damage to cranial nerve X, which supplies the
heart mainly from the right
D. To avoid injuring the thoracic duct
E. Less chance of vocal cord paralysis and hoarseness
from the left

A

A. To avoid injuring the recurrent laryngeal nerve, which
follows a more torturous route on the right
B. To avoid damage to the dominant superior laryngeal
nerve on the right
C. To avoid damage to cranial nerve X, which supplies the
heart mainly from the right

D. To avoid injuring the thoracic duct
E. Less chance of vocal cord paralysis and hoarseness
from the left

Vagal nerve stimulation must be performed on the left
side so that the cardiac innervation of CN X is unaffected
(Youmans, pp. 2644-2645).

How well did you know this?
1
Not at all
2
3
4
5
Perfectly
25
Q

What is the treatment of choice for chronic, intractable
brachial plexus avulsion injury?
A. Cordotomy
B. Dorsal root entry zone (DREZ) lesioning
C. Morphine pump placement
D. Midline myelotomy
E. Ventroposterior lateral (VPL) thalamic deep brain
stimulation

A

A. Cordotomy
**B. Dorsal root entry zone (DREZ) lesioning **
C. Morphine pump placement
D. Midline myelotomy
E. Ventroposterior lateral (VPL) thalamic deep brain
stimulation

Dorsal root entry zone (DREZ) lesioning involves
radiofrequency ablation along the dorsolateral sulcus of the
spinal cord. The DREZ procedure is most effective in the
treatment of brachial and lumbar plexus avulsion pain.
Direct sectioning of the spinothalamiC tract (cordotomy) is
very effective for unilateral pain below the upper chest
region; however, it is associa ted wi th many complica tions
and is usually performed only in terminally ill patients.
Complications of cordotomy include hemiparesis, respiratory depreSSion (“Ondine’s curse” with bilateral procedures),
and dysesthesias. Midline mye!otomies can also be performed
to interrupt the decussating fibers of the spinothalamic tract,
and this can be quite effective in the treatment of chronic
pelvic pain secondary to cancer. Intrathecal narcotic administration is typically used for the treatment of chronic pain
associated with malignancy or failed low back syndrome.
Deep brain stimulation of the VPL and VPM nuclei of the
thalamus as well as the periaqueductal gray have been performed in the treatment of thalamic pain states, postherpetic
neuralgia, and causalgia. Chronic low-threshold stimulation
of the motor cortex is also utilized in the treatment of thalamic pain syndromes; it is thought to work by retrograde
thalamic stimulation pathways (Kaye and Blacl<, pp. 1521-
1537; Greenberg, pp. 365-370; Youmans, pp. 3025- 3030,
3045-3048, 3068- 3070, 3101, 3125, 3128-3129; Will<ins,
pp. 4036-4038, 4055- 4059)

How well did you know this?
1
Not at all
2
3
4
5
Perfectly
26
Q

What basal cistern(s) contain portions of the vein of
Rosenthal?
1. Crural
2. Quadrigeminal
3. Ambient
4. Quadrigeminal

A. 1,2, and 3 are correct
B. 1 and 3 are correct
C. 2 and 4 are correct
D. Only 4 is correct
E. All of the above

A

A. 1,2, and 3 are correct
**B. 1 and 3 are correct **
C. 2 and 4 are correct
D. Only 4 is correct
E. All of the above

The ambient and crural cisterns contain portions of
the basal vein of Rosenthal (Youmans, pp. 36- 39).

How well did you know this?
1
Not at all
2
3
4
5
Perfectly
27
Q

A 3-month-old boy is brought to your office for an
abnormally shaped head. The child is noted to have a flat
occiput on the left, a left ear that is anterior to the right, and
a prominent forehead and malar eminence on the left. ‘Vhat
is the most likely etiology of this deformity?

A. Left lambdoid synostosis
B. Right lambdoid synostosis
C. Sagittal suture synostosis
D. Skull molding
E. Right coronal suture synostosis

A

A. Left lambdoid synostosis
B. Right lambdoid synostosis
C. Sagittal suture synostosis
**D. Skull molding **
E. Right coronal suture synostosis

Lambdoid synostosis is among the rarest forms of
suture synostosis, while sagittal synostosis is the most common. After the American Academy of Pediatrics published its
recommendations that all children sleep on their back, the
incidence of skull molding increased. Most infants sleep on
their backs and spend the rest of the day sitting in a car seat
or infant seat. Children with this condition are often noted to
have a flat occiput, one ear that is anterior to the other in
an axial plane, and a prominent forehead and malar eminence. Many infants may also have a mild torticollis due to a
shortened sternocleidomastoid muscle on one side, as well as
decreased range of motion in the neck. The skull deformity
usually responds very well to behavioral modification, which
includes having the parents turn the infant or child from side
to side during sleep and reducing the amount of time spent in
a car seat. If this is unsuccessful, a molding helmet or band
is often helpful. For nonresponders, a variety of occipital
remolding surgical procedures are available (Committee on
Education in Neurological Surgery, pp. 43, 137; Pattisapu,
pp.178- 179)

How well did you know this?
1
Not at all
2
3
4
5
Perfectly
28
Q

All of the following are suboptimal conditions for placement of an odontoid screw EXCEPT

A. Old fractures (> 6 weeks)
B. Diagonal fractures through the odontoid process
C. Barrel-chested patient
D. Odontoid fracture that is displaced anteriorly
E. An intact transverse ligament

A

A. Old fractures (> 6 weeks)
B. Diagonal fractures through the odontoid process
C. Barrel-chested patient
D. Odontoid fracture that is displaced anteriorly
E. An intact transverse ligament

An intact transverse ligament must be confirmed
preoperatively prior to placement of an odontoid screw. Old
fractures in which a nonunion has already formed, diagonal
fractures through the odontoid process, and comminuted
odontoid fractures do not aUow for proper odontoid screw
placement due to suboptimal arthrodesis rates and inadequate screw purchase and compression effects (Kaye and
Blacl<, pp. 2048-2050; Youmans, pp. 4943- 4945; Benzel,
pp.225- 228).

How well did you know this?
1
Not at all
2
3
4
5
Perfectly
29
Q

A 56-year-old female underwent clipping of the
aneurysm depicted on the angiogram below (Figure 6.29Q).
Upon awakening from surgery, she was noted to have greater
wealmess in her left arm than in her left leg. What is the most
likely reason for this new deficit?

A. Injury of a blood vessel originating from the A2 segment of the anterior cerebral artery
B. Venous infarct from excessive frontal lobe retraction
C. Injury to the small perforating blood vessels originating from the anterior communicating artery
D. Posterior internal capsule infarction from microemboli
originating from the internal carotid artery
E. Mesial temporal lobe retraction

A

**A. Injury of a blood vessel originating from the A2 segment of the anterior cerebral artery **
B. Venous infarct from excessive frontal lobe retraction
C. Injury to the small perforating blood vessels originating from the anterior communicating artery
D. Posterior internal capsule infarction from microemboli
originating from the internal carotid artery
E. Mesial temporal lobe retraction

A. The recurrent artery of Heubner usually originates
from the A2 segment of the anterior cerebral artery (ACA).
Injury of this vessel during surgery is generally witnessed on
the first postoperative day, as evidenced by a hypodensity on
CT scan. Although injury to this vessel during surgery often
presents with arm wealmess greater than leg wealmess on
the nondominant side and cognitive deficits from injury on
the dominant Side, a percentage of patients do not have
any adverse neurologic sequelae after injury of this vessel.
Identifying this vessel early in the dissection may prevent
later injury (Greenberg, pp. 103-104).

How well did you know this?
1
Not at all
2
3
4
5
Perfectly
30
Q

Basilar skull fractures can be associated with cranial
nerve palsies, bilateral periorbital ecchymosis, mastoid
ecchymosis, hemotympanum, and rhinorrhea. Nasal
drainage that is not clearly CSF can be assayed for
____ , which is unique to CSF and ____ _

A. a-Fetoprotein, saliva
B. Transferrin, vitreous fluid of the eye
C. ~2- Transferrin, tears
D. Hypoglycorrhachia, nasal secretions
E. Sodium, peritoneal fluid

A

A. a-Fetoprotein, saliva
**B. Transferrin, vitreous fluid of the eye **
C. ~2- Transferrin, tears
D. Hypoglycorrhachia, nasal secretions
E. Sodium, peritoneal fluid

Prtransferrin is present in the CSF but absent in the
tears, saliva, peritoneal fluid , nasal exudates, and serum
(except for newborns or those with liver disease). The
only other source is the vitreous humor of the eye. Other
commonly employed tests include measuring the glucose
level of the fluid (CSF glucose> 30 mg %, whereas lacrimal and
mucous secretions are < 5 mg%) or placing the fluid on a
piece of linen (bed sheet, pillowcase) and seeing whether a
ring of blood surrounded by a larger concentric ring of clear
fluid develops (“ring” or “halo” sign) (Greenberg, pp. 168-
169).

How well did you know this?
1
Not at all
2
3
4
5
Perfectly
31
Q

A 4-month-old male fell from his crib and suffered a
growing skull fracture . All of the following are true of this
disease entity EXCEPT?

A. May be assoc iated with late neurologic deficits
B. A dural laceration is always present
C. There may be ongoing damage to underlying brain
from continued herniation of brain through the defect
D. CSF diversion is often the only treatment required for
this fracture pattern
E. May be associated with leptomeningeal cyst development

A

A. May be assoc iated with late neurologic deficits
B. A dural laceration is always present
C. There may be ongoing damage to underlying brain
from continued herniation of brain through the defect
**D. CSF diversion is often the only treatment required for
this fracture pattern **
E. May be associated with leptomeningeal cyst development

Growing skull fractures occur in children, are always
associated with underlying dural lacerations, and almost
always require surgical treatment. Early surgical correction
of growing fractures is often necessary because these types of
fractures almost never heal spontaneously and late neuro logic deterioration can occur. The pathophysiology requires
a fracture with enough force to cause a tear in the dura
(which is always present) and an underlying constant
force such as a growing brain, leptomeningeal cyst, hydrocephalus, or porencephaly. The precise etiology of late
neurologic deficits remains unclear but is believed to occur
by one of two mechanisms. There may be ongoing brain
damage from brain herniation and pulsations of the brain
against the bone edges. Alternatively, some have proposed
that there may be vascular compromise of blood vessels at
the bone edges. Although some authors have recommended a
CSF diversion procedure for growing skull fractures, direct
repair of the fracture is the definitive treatment (Youmans,
pp. 3468-3469; Will<ins, pp. 2757- 2761)

How well did you know this?
1
Not at all
2
3
4
5
Perfectly
32
Q

A 54-year-old female completed radiation therapy for
breast cancer. She has been complaining of wealmess in
her left arm over the past 3 months and is concerned there
may be recurrence of her cancer. How can her physician
distinguish between radiation-induced plexopathy and
cancerous invasion of the brachial plexus ?

A. Radiation-induced plexopathy is frequently accompanied by pain and lack of edema
B. Cancerous invasion of the brachial plexus is accompanied by lymphedema, painless wealmess, and
sensory loss
C. Radiation-induced plexopathy is frequently reversible
D. l’vlyokymia on EMG favors radiation-induced plexopathy
E. Prolonged I-I latency is typically seen only with brachial
plexopathy secondary to radiation damage

A

A. Radiation-induced plexopathy is frequently accompanied by pain and lack of edema
B. Cancerous invasion of the brachial plexus is accompanied by lymphedema, painless wealmess, and
sensory loss
C. Radiation-induced plexopathy is frequently reversible
**D. l’vlyokymia on EMG favors radiation-induced plexopathy **
E. Prolonged I-I latency is typically seen only with brachial
plexopathy secondary to radiation damage

Myokymia (quivering of muscles) on EMG strongly
favors radiation-induced changes of the brachial plexus.
Lymphedema, painless paresis, and numbness suggest
radiation-induced injury (irreversible), whereas pain and
lack of edema suggest recurrent tumor. When ancillary
studies are inconclusive, surgical exploration of the plexus
may have to be considered (Merritt, p. 459)

How well did you know this?
1
Not at all
2
3
4
5
Perfectly
33
Q

A 42-year-old male falls 25 feet while at work and
arrives at the emergency department with a Glasgow Coma
Scale (GCS) score of 5, a dilated and nonreactive right
pupil, and a mean arterial blood pressure of 80. IVter airway
management and fluid resuscitation, his GCS improves to 7,
but his right hemiparesis and nonreactive pupil remain
unchanged. The patient also sustained a pelvic fracture, a left
humeral fracture , splenic and liver lacerations, and multiple
fractures of the cervical spine.

Initial management of this patient should include
1. Begin hyperventilation to decrease the pC02
2. Administer mannitol on arrival to the emergency
department because of clinical evidence of an asymmetric exam
3. Complete the primary survey, obtain cervical spine filins
and chest x-ray, and then move directly to CT scan
4. The patient should be started on pentobarbital for
elevated intracranial pressure immediately after
completion of the primary survey if no mass lesion
is found on CT

A. 1,2, and3 are correct
B. 1 and 3 are correct
C. 2 and 4 are correct
D. Only 4 is correct
E. All of the above

A

A. 1,2, and3 are correct
**B. 1 and 3 are correct **
C. 2 and 4 are correct
D. Only 4 is correct
E. All of the above

Current head injury research guidelines suggest that mannitol and hyperventilation may
exacerbate cerebral ischemia after head injury. However,
mannitol and hyperventilation are recommended for those
patients with acute head injury as a temporary measure
to control elevated intracranial pressure. Hyperventilation
may be commenced immediately, but mannitol should be
withheld until the primary survey is complete and adequate
intravascular volume and urine output are realized. The
mechanism of action of mannitol is still debated, but it is
believed to have the follOWing beneficial effects. First, it acts
to immediately expand the plasma by reducing hematocrit
and blood viscosity (improved rheology), which improves
CBF and O2 delivery. This reduces ICP very quicldy, which is
most marked in patients with CPP < 70 mm r-rg. Second, it
draws edema from the adjacent cerebral parenchyma into
the intravascular compartment (osmotic effect), an effect
that may last up to 6 hours. And last, it is a possible freeradical scavenger. Despite these short-lived but beneficial
effects, the administration of mannitol is not without risks. It
opens the blood-brain barrier and can potentially draw fluid
into the CNS and aggravate vasogenic cerebral edema. It
should also be used cautiously with concomitant administration of steroids and phenytoin (Dilantin), as it may cause a
nonketotic hyperosmolar state, with high mortality. With
overuse, it can also result in hypertension and increased
CBF if cerebral autoregulation is defective . . And last, high
doses carry a significant risk for the development of acute
renal failure , especially with coexisting sepsis, concomitant nephrotoxic drug use, serum osmolarity> 320, or preexisting kidney disease. Some patients with mass lesions on
one side may not uniformly develop contralateral hemiparesis

How well did you know this?
1
Not at all
2
3
4
5
Perfectly
34
Q

A 42-year-old male falls 25 feet while at work and
arrives at the emergency department with a Glasgow Coma
Scale (GCS) score of 5, a dilated and nonreactive right
pupil, and a mean arterial blood pressure of 80. IVter airway
management and fluid resuscitation, his GCS improves to 7,
but his right hemiparesis and nonreactive pupil remain
unchanged. The patient also sustained a pelvic fracture, a left
humeral fracture , splenic and liver lacerations, and multiple
fractures of the cervical spine.

The CT scan of the brain is depicted below (Figure
6.34Q). Why did this patient develop hemiparesis on the
same side as the hematoma ?

A. Shift of the brainstem away frol11 the mass producing
compression of the contralateral cerebral peduncle
against the tentorium
B. The patient lil{ely suffered a Duret hemorrhage
C. There was likely a contusion in the underlying motor
cortex on the contralateral side that was not detected
on the initial CT scan
D. The patient likely had a left internal ca rotid artery
dissection that subsequently showered emboli to the
distal vasculature
E. There was an associated fracture of the transverse
foramen on the left, which produced a vertebral artery
dissection and small infarct in the ventral pons

A

**A. Shift of the brainstem away frol11 the mass producing
compression of the contralateral cerebral peduncle
against the tentorium **
B. The patient lil{ely suffered a Duret hemorrhage
C. There was likely a contusion in the underlying motor
cortex on the contralateral side that was not detected
on the initial CT scan
D. The patient likely had a left internal ca rotid artery
dissection that subsequently showered emboli to the
distal vasculature
E. There was an associated fracture of the transverse
foramen on the left, which produced a vertebral artery
dissection and small infarct in the ventral pons

Current head injury research guidelines suggest that mannitol and hyperventilation may
exacerbate cerebral ischemia after head injury. However,
mannitol and hyperventilation are recommended for those
patients with acute head injury as a temporary measure
to control elevated intracranial pressure. Hyperventilation
may be commenced immediately, but mannitol should be
withheld until the primary survey is complete and adequate
intravascular volume and urine output are realized. The
mechanism of action of mannitol is still debated, but it is
believed to have the follOWing beneficial effects. First, it acts
to immediately expand the plasma by reducing hematocrit
and blood viscosity (improved rheology), which improves
CBF and O2 delivery. This reduces ICP very quicldy, which is
most marked in patients with CPP < 70 mm r-rg. Second, it
draws edema from the adjacent cerebral parenchyma into
the intravascular compartment (osmotic effect), an effect
that may last up to 6 hours. And last, it is a possible freeradical scavenger. Despite these short-lived but beneficial
effects, the administration of mannitol is not without risks. It
opens the blood-brain barrier and can potentially draw fluid
into the CNS and aggravate vasogenic cerebral edema. It
should also be used cautiously with concomitant administration of steroids and phenytoin (Dilantin), as it may cause a
nonketotic hyperosmolar state, with high mortality. With
overuse, it can also result in hypertension and increased
CBF if cerebral autoregulation is defective . . And last, high
doses carry a significant risk for the development of acute
renal failure , especially with coexisting sepsis, concomitant nephrotoxic drug use, serum osmolarity> 320, or preexisting kidney disease. Some patients with mass lesions on
one side may not uniformly develop contralateral hemiparesis

How well did you know this?
1
Not at all
2
3
4
5
Perfectly
35
Q

A 42-year-old male falls 25 feet while at work and
arrives at the emergency department with a Glasgow Coma
Scale (GCS) score of 5, a dilated and nonreactive right
pupil, and a mean arterial blood pressure of 80. IVter airway
management and fluid resuscitation, his GCS improves to 7,
but his right hemiparesis and nonreactive pupil remain
unchanged. The patient also sustained a pelvic fracture, a left
humeral fracture , splenic and liver lacerations, and multiple
fractures of the cervical spine.

‘Vhich of the following are possible complications of
mannitol administration?
1. Aggravation of vasogenic edema
2. Development of a hyperosmolar nonketotic state
3. Acute tubular necrosis
4. Hypotension

A. 1,2, and 3 are correct
B. 1 and 3 are correct
C. 2 and 4 are correct
D. Only 4 is correct
E. All of the above

A

A. 1,2, and 3 are correct
B. 1 and 3 are correct
C. 2 and 4 are correct
D. Only 4 is correct
**E. All of the above **

Current head injury research guidelines suggest that mannitol and hyperventilation may
exacerbate cerebral ischemia after head injury. However,
mannitol and hyperventilation are recommended for those
patients with acute head injury as a temporary measure
to control elevated intracranial pressure. Hyperventilation
may be commenced immediately, but mannitol should be
withheld until the primary survey is complete and adequate
intravascular volume and urine output are realized. The
mechanism of action of mannitol is still debated, but it is
believed to have the follOWing beneficial effects. First, it acts
to immediately expand the plasma by reducing hematocrit
and blood viscosity (improved rheology), which improves
CBF and O2 delivery. This reduces ICP very quicldy, which is
most marked in patients with CPP < 70 mm r-rg. Second, it
draws edema from the adjacent cerebral parenchyma into
the intravascular compartment (osmotic effect), an effect
that may last up to 6 hours. And last, it is a possible freeradical scavenger. Despite these short-lived but beneficial
effects, the administration of mannitol is not without risks. It
opens the blood-brain barrier and can potentially draw fluid
into the CNS and aggravate vasogenic cerebral edema. It
should also be used cautiously with concomitant administration of steroids and phenytoin (Dilantin), as it may cause a
nonketotic hyperosmolar state, with high mortality. With
overuse, it can also result in hypertension and increased
CBF if cerebral autoregulation is defective . . And last, high
doses carry a significant risk for the development of acute
renal failure , especially with coexisting sepsis, concomitant nephrotoxic drug use, serum osmolarity> 320, or preexisting kidney disease. Some patients with mass lesions on
one side may not uniformly develop contralateral hemiparesis

How well did you know this?
1
Not at all
2
3
4
5
Perfectly
36
Q

A 42-year-old male falls 25 feet while at work and
arrives at the emergency department with a Glasgow Coma
Scale (GCS) score of 5, a dilated and nonreactive right
pupil, and a mean arterial blood pressure of 80. IVter airway
management and fluid resuscitation, his GCS improves to 7,
but his right hemiparesis and nonreactive pupil remain
unchanged. The patient also sustained a pelvic fracture, a left
humeral fracture , splenic and liver lacerations, and multiple
fractures of the cervical spine.

After surgery for evacuation of the right subdural
hematoma, CT angiogram was obtained to rule out a vertebral artery injury because of the multiple fractures of the
cervical spine extending through the transverse foramina .
The study was inconclusive, and a follow-up angiogram (Figure 6.36Q) was obtained later that evening after hematoma
evacuation. What would be the most reasonable treatment
strategy at this point for this multisystem trauma patient?
Consider that the patient adequately fills the posterior circulation from the right vertebral artery.
1. Commencement of a heparin infusion with a goal of
keeping the PTT approximately two times the normal
level
2. Antiplatelet therapy
3. Intravenous t-PA
4. Endovascular sacrifice of the occluded vertebral artery

A. 1,2, and 3 are correct
B. 1 and 3 are correct
C. 2 and 4 are correct
D. Only 4 is correct
E. All of the above

A

A. 1,2, and 3 are correct
B. 1 and 3 are correct
**C. 2 and 4 are correct **
D. Only 4 is correct
E. All of the above

Current head injury research guidelines suggest that mannitol and hyperventilation may
exacerbate cerebral ischemia after head injury. However,
mannitol and hyperventilation are recommended for those
patients with acute head injury as a temporary measure
to control elevated intracranial pressure. Hyperventilation
may be commenced immediately, but mannitol should be
withheld until the primary survey is complete and adequate
intravascular volume and urine output are realized. The
mechanism of action of mannitol is still debated, but it is
believed to have the follOWing beneficial effects. First, it acts
to immediately expand the plasma by reducing hematocrit
and blood viscosity (improved rheology), which improves
CBF and O2 delivery. This reduces ICP very quicldy, which is
most marked in patients with CPP < 70 mm r-rg. Second, it
draws edema from the adjacent cerebral parenchyma into
the intravascular compartment (osmotic effect), an effect
that may last up to 6 hours. And last, it is a possible freeradical scavenger. Despite these short-lived but beneficial
effects, the administration of mannitol is not without risks. It
opens the blood-brain barrier and can potentially draw fluid
into the CNS and aggravate vasogenic cerebral edema. It
should also be used cautiously with concomitant administration of steroids and phenytoin (Dilantin), as it may cause a
nonketotic hyperosmolar state, with high mortality. With
overuse, it can also result in hypertension and increased
CBF if cerebral autoregulation is defective . . And last, high
doses carry a significant risk for the development of acute
renal failure , especially with coexisting sepsis, concomitant nephrotoxic drug use, serum osmolarity> 320, or preexisting kidney disease. Some patients with mass lesions on
one side may not uniformly develop contralateral hemiparesis

How well did you know this?
1
Not at all
2
3
4
5
Perfectly
37
Q

A 45-year-old male presents to an emergency
room with fever, nausea, vomiting, and severe headache.
CT of the brain is normal. Lumbar puncture reveals slightly
elevated red blood cells, but normal protein, glucose, white
blood cell count, and no xanthochromia. His angiogram is
depicted below.

What is the most likely etiology of the abnormality
depicted in the angiogram below?

A. Head trauma
B. Infection
C. Genetic predisposition
D. Collagen vascular disease
E. Hypertension

A

A. Head trauma
**B. Infection **
C. Genetic predisposition
D. Collagen vascular disease
E. Hypertension

Mycotic or infective aneurysms account for
about 4% of intracranial aneurysms and are most commonly
found along the distribution of the distal middle cerebral
artery (MCA). Patients with head trauma also are at risk of
developing aneurysms, although these are most frequently
located along the distal anterior cerebral artery distribution.
Mycotic aneurysms occur in about 3 to 15% of patients with
subacute bacte rial endoca rditis (SBE), and the most com1110n organism isolated from the blood is Streptococclis
viTidans species. Patients that are IV drug abusers or
immunodeficient (HIV) are at increased risk of developing

How well did you know this?
1
Not at all
2
3
4
5
Perfectly
38
Q

A 45-year-old male presents to an emergency
room with fever, nausea, vomiting, and severe headache.
CT of the brain is normal. Lumbar puncture reveals slightly
elevated red blood cells, but normal protein, glucose, white
blood cell count, and no xanthochromia. His angiogram is
depicted below.

This finding occurs most frequently in what condition?
A. Alcoholism
B. Ehlers-Danlos disease
C. Subacute bacterial endocarditis
D. Marfan’s syndrome
E. Polycystic kidney disease

A

A. Alcoholism
B. Ehlers-Danlos disease
C. Subacute bacterial endocarditis
D. Marfan’s syndrome
E. Polycystic kidney disease

Mycotic or infective aneurysms account for
about 4% of intracranial aneurysms and are most commonly
found along the distribution of the distal middle cerebral
artery (MCA). Patients with head trauma also are at risk of
developing aneurysms, although these are most frequently
located along the distal anterior cerebral artery distribution.
Mycotic aneurysms occur in about 3 to 15% of patients with
subacute bacte rial endoca rditis (SBE), and the most com1110n organism isolated from the blood is Streptococclis
viTidans species. Patients that are IV drug abusers or
immunodeficient (HIV) are at increased risk of developing

How well did you know this?
1
Not at all
2
3
4
5
Perfectly
39
Q

A 45-year-old male presents to an emergency
room with fever, nausea, vomiting, and severe headache.
CT of the brain is normal. Lumbar puncture reveals slightly
elevated red blood cells, but normal protein, glucose, white
blood cell count, and no xanthochromia. His angiogram is
depicted below.

How should this problem be treated?
A. Observation followed by repeat angiography in 6 months
B. Antibiotics followed by repeat angiography
C. Emergent surgery
D. Stentlcoiling followed by blood pressure control
E. Steroids

A

A. Observation followed by repeat angiography in 6 months
**B. Antibiotics followed by repeat angiography **
C. Emergent surgery
D. Stentlcoiling followed by blood pressure control
E. Steroids

Mycotic or infective aneurysms account for
about 4% of intracranial aneurysms and are most commonly
found along the distribution of the distal middle cerebral
artery (MCA). Patients with head trauma also are at risk of
developing aneurysms, although these are most frequently
located along the distal anterior cerebral artery distribution.
Mycotic aneurysms occur in about 3 to 15% of patients with
subacute bacte rial endoca rditis (SBE), and the most com1110n organism isolated from the blood is Streptococclis
viTidans species. Patients that are IV drug abusers or
immunodeficient (HIV) are at increased risk of developing

How well did you know this?
1
Not at all
2
3
4
5
Perfectly
40
Q

A lS-month-old girl was brought to the emergency
department for lethargy, nausea, and vomiting and was found
to have aqueductal stenosis on brain MRI.

What is the best treatment strategy for this patient?
A. Observation
B. Placement of a subgaleal shunt
C. Placement of a ventriculoperitoneal shunt followed
by endoscopic third ventriculostomy if shunting fails
D. EndoscopiC third ventriculostomy
E. EndoscopiC third ventriculostomy followed by septostomy

A

A. Observation
B. Placement of a subgaleal shunt
C. Placement of a ventriculoperitoneal shunt followed
by endoscopic third ventriculostomy if shunting fails
**D. EndoscopiC third ventriculostomy **
E. EndoscopiC third ventriculostomy followed by septostomy

ETV is a commonly performed procedure for patients with aqueductal stenosis (AS). Although
there is some controversy about the age at which this
procedure should first be employed, results indicate high
success rates for properly selected patients. Complications of
shunting may include slit ventricle syndrome, intracranial
hypotension, subdural hematomas, craniosynostosis, microcephaly, and overdrainage, which are typically not noted
after endoscopy. The precise location to fenestrate the floor
of the third ventricle may vary on a case-by-case baSiS, but
perforating the floor anterior to the mamillary bodies and
posterior to the infundibulum seems to a popular approach .
Performing a septostomy in conjunction to a third ventriculostomy does not improve results in patients with AS, as
the obstruction is downstream to the foramen of Monro. A
patient with scarring or a cyst obstructing one foramen of
Monro would likely benefit from this ancillary procedure.
Predicting the success rate of ETV by preoperative imaging
studies has proven to be very difficult, although identifying
relevant anatomy (thickness of the floor of the third
ventricle, location of basilar artery) to help guide the operation has proven to be effective (Kaye and Blacl~, pp. 789- 797;
Youmans, pp. 3429- 3430; Will~ins, pp. 541-542).

How well did you know this?
1
Not at all
2
3
4
5
Perfectly
41
Q

A lS-month-old girl was brought to the emergency
department for lethargy, nausea, and vomiting and was found
to have aqueductal stenosis on brain MRI.

All of the following are advantages of endoscopic third
ventriculostomy (ETV) over shunting EXCEPT?
A. Lower rate of subdural hematoma formation with ETV
B. Higher rate of craniosynostosis with ETV
C. Lower infection rate with ETV
D. Physiologic CSF diversion with ETV
E. Higher chance of overdrainage with shunt placement

A

A. Lower rate of subdural hematoma formation with ETV
**B. Higher rate of craniosynostosis with ETV **
C. Lower infection rate with ETV
D. Physiologic CSF diversion with ETV
E. Higher chance of overdrainage with shunt placement

ETV is a commonly performed procedure for patients with aqueductal stenosis (AS). Although
there is some controversy about the age at which this
procedure should first be employed, results indicate high
success rates for properly selected patients. Complications of
shunting may include slit ventricle syndrome, intracranial
hypotension, subdural hematomas, craniosynostosis, microcephaly, and overdrainage, which are typically not noted
after endoscopy. The precise location to fenestrate the floor
of the third ventricle may vary on a case-by-case baSiS, but
perforating the floor anterior to the mamillary bodies and
posterior to the infundibulum seems to a popular approach .
Performing a septostomy in conjunction to a third ventriculostomy does not improve results in patients with AS, as
the obstruction is downstream to the foramen of Monro. A
patient with scarring or a cyst obstructing one foramen of
Monro would likely benefit from this ancillary procedure.
Predicting the success rate of ETV by preoperative imaging
studies has proven to be very difficult, although identifying
relevant anatomy (thickness of the floor of the third
ventricle, location of basilar artery) to help guide the operation has proven to be effective (Kaye and Blacl~, pp. 789- 797;
Youmans, pp. 3429- 3430; Will~ins, pp. 541-542).

42
Q

A lS-month-old girl was brought to the emergency
department for lethargy, nausea, and vomiting and was found
to have aqueductal stenosis on brain MRI.

All of the following are true about preoperative planning
for ETV EXCEPT?
A. It is relatively straightforward to accurately determine
the future function of the subarachnoid pathways and
patency of the ETV as long as a high resolution MR
cisternogram is obtained preoperatively that identifies
the level of the block
B. MRI can accurately delineate the anatomy of the foramen oOdonro, third ventricle, and massa intermedia
C. The position of the basilar artery and the thiclmess
of the third ventricular floor can be verified on most
preoperative MRIs
D. A prior history of CSF infection may decrease the
success rate of ETV
E. A prior history of a shunt is not an absolute contraindication for ETV

A

**A. It is relatively straightforward to accurately determine
the future function of the subarachnoid pathways and
patency of the ETV as long as a high resolution MR
cisternogram is obtained preoperatively that identifies
the level of the block **
B. MRI can accurately delineate the anatomy of the foramen oOdonro, third ventricle, and massa intermedia
C. The position of the basilar artery and the thiclmess
of the third ventricular floor can be verified on most
preoperative MRIs
D. A prior history of CSF infection may decrease the
success rate of ETV
E. A prior history of a shunt is not an absolute contraindication for ETV

ETV is a commonly performed procedure for patients with aqueductal stenosis (AS). Although
there is some controversy about the age at which this
procedure should first be employed, results indicate high
success rates for properly selected patients. Complications of
shunting may include slit ventricle syndrome, intracranial
hypotension, subdural hematomas, craniosynostosis, microcephaly, and overdrainage, which are typically not noted
after endoscopy. The precise location to fenestrate the floor
of the third ventricle may vary on a case-by-case baSiS, but
perforating the floor anterior to the mamillary bodies and
posterior to the infundibulum seems to a popular approach .
Performing a septostomy in conjunction to a third ventriculostomy does not improve results in patients with AS, as
the obstruction is downstream to the foramen of Monro. A
patient with scarring or a cyst obstructing one foramen of
Monro would likely benefit from this ancillary procedure.
Predicting the success rate of ETV by preoperative imaging
studies has proven to be very difficult, although identifying
relevant anatomy (thickness of the floor of the third
ventricle, location of basilar artery) to help guide the operation has proven to be effective (Kaye and Blacl~, pp. 789- 797;
Youmans, pp. 3429- 3430; Will~ins, pp. 541-542).

43
Q

A lS-month-old girl was brought to the emergency
department for lethargy, nausea, and vomiting and was found
to have aqueductal stenosis on brain MRI.

What is the optimal site for fenestrating the floor of the
third ventricle during ETV?
A. Posterior to the mammillary bodies
B. Anterior to the infundibular recess, posterior to the
prechiasmatic space
C. In the most translucent area of the floor of the third
ventricle
D. Anterior to the mammillary bodies, posterior to the
infundibular recess
E. Anterior to the pulsations of the basilar artery

A

A. Posterior to the mammillary bodies
B. Anterior to the infundibular recess, posterior to the
prechiasmatic space
C. In the most translucent area of the floor of the third
ventricle
**D. Anterior to the mammillary bodies, posterior to the
infundibular recess **
E. Anterior to the pulsations of the basilar artery

ETV is a commonly performed procedure for patients with aqueductal stenosis (AS). Although
there is some controversy about the age at which this
procedure should first be employed, results indicate high
success rates for properly selected patients. Complications of
shunting may include slit ventricle syndrome, intracranial
hypotension, subdural hematomas, craniosynostosis, microcephaly, and overdrainage, which are typically not noted
after endoscopy. The precise location to fenestrate the floor
of the third ventricle may vary on a case-by-case baSiS, but
perforating the floor anterior to the mamillary bodies and
posterior to the infundibulum seems to a popular approach .
Performing a septostomy in conjunction to a third ventriculostomy does not improve results in patients with AS, as
the obstruction is downstream to the foramen of Monro. A
patient with scarring or a cyst obstructing one foramen of
Monro would likely benefit from this ancillary procedure.
Predicting the success rate of ETV by preoperative imaging
studies has proven to be very difficult, although identifying
relevant anatomy (thickness of the floor of the third
ventricle, location of basilar artery) to help guide the operation has proven to be effective (Kaye and Blacl~, pp. 789- 797;
Youmans, pp. 3429- 3430; Will~ins, pp. 541-542).

44
Q

The borders of the lateral recess include all of the followingEXCEPT?
A. Pedicle
B. Superior articular facet
C. Inferior articular facet
D. Vertebral body
E. Spinal canal/thecal sac

A

A. Pedicle
B. Superior articular facet
**C. Inferior articular facet **
D. Vertebral body
E. Spinal canal/thecal sac

Compression of nerve roots in the
lateral recess (lateral recess syndrome) can occur between
a hypertrophied superior articular facet (dorsally), the
pedicle (laterally), and the inferior vertebral body (ventrally). Medially, the lateral recess opens toward the spinal
canal/thecal sac. The characteristic feature of lateral recess
syndrome is that of radicular symptoms that occur mainly
when the patient is walking or standing and are relieved
by Sitting, squatting forward, lying on either side, andlor
postures that accentuate lumbar kyphosis. This is opposite
to what is seen with patients harboring discogenic disease ,
who are uncomfortable while sitting. With the lateral recess
syndrome, adequate decompression involves laminectomy
with resec tion of the medial third of the hypertrophied facet
(media l facetectomy), which is usually the superior articular
process (Wi ll in s, pp. 3841- 3845).

45
Q

The underlying cause of lateral recess stenosis is osteophyte formation originating from what structur
A. Inferior articular process
B. Pedicle
C. Superior articular process
D. Ligamentum flavum hypertrophy
E. Vertebral body

A

A. Inferior articular process
B. Pedicle
**C. Superior articular process **
D. Ligamentum flavum hypertrophy
E. Vertebral body

Compression of nerve roots in the
lateral recess (lateral recess syndrome) can occur between
a hypertrophied superior articular facet (dorsally), the
pedicle (laterally), and the inferior vertebral body (ventrally). Medially, the lateral recess opens toward the spinal
canal/thecal sac. The characteristic feature of lateral recess
syndrome is that of radicular symptoms that occur mainly
when the patient is walking or standing and are relieved
by Sitting, squatting forward, lying on either side, andlor
postures that accentuate lumbar kyphosis. This is opposite
to what is seen with patients harboring discogenic disease ,
who are uncomfortable while sitting. With the lateral recess
syndrome, adequate decompression involves laminectomy
with resec tion of the medial third of the hypertrophied facet
(media l facetectomy), which is usually the superior articular
process (Wi ll in s, pp. 3841- 3845).

46
Q

Although quite similar to the symptoms of radiculopathy
secondary to discogenic disease, lateral recess stenosis can
be differentiated from discogenic disease by which of the
following?
A. Pain in the lateral recess syndrome is exacerbated by
walking or standing
B. Failure of coughing or sneezing to aggravate pain in
discogenic disease
C. Positive straight leg raising in lateral recess syndrome
D. Pain in lateral recess syndrome is relieved by postures
accentuating lumbar lordosis
E. There is a slightly higher incidence of bladder incontinence with lateral recess stenosis

A

A. Pain in the lateral recess syndrome is exacerbated by
walking or standing

B. Failure of coughing or sneezing to aggravate pain in
discogenic disease
C. Positive straight leg raising in lateral recess syndrome
D. Pain in lateral recess syndrome is relieved by postures
accentuating lumbar lordosis
E. There is a slightly higher incidence of bladder incontinence with lateral recess stenosis

Compression of nerve roots in the
lateral recess (lateral recess syndrome) can occur between
a hypertrophied superior articular facet (dorsally), the
pedicle (laterally), and the inferior vertebral body (ventrally). Medially, the lateral recess opens toward the spinal
canal/thecal sac. The characteristic feature of lateral recess
syndrome is that of radicular symptoms that occur mainly
when the patient is walking or standing and are relieved
by Sitting, squatting forward, lying on either side, andlor
postures that accentuate lumbar kyphosis. This is opposite
to what is seen with patients harboring discogenic disease ,
who are uncomfortable while sitting. With the lateral recess
syndrome, adequate decompression involves laminectomy
with resec tion of the medial third of the hypertrophied facet
(media l facetectomy), which is usually the superior articular
process (Wi ll in s, pp. 3841- 3845).

47
Q

\That is the best surgical strategy for patients with lateral
recess stenosis?
A. Laminectomy
B. Laminectomy with resection of the medial third of the
hypertrophied facet (medial facetectomy)
C. Microdiscectomy
D. Laminectomy and fusion
E. None of the above

A

A. Laminectomy
**B. Laminectomy with resection of the medial third of the
hypertrophied facet (medial facetectomy) **
C. Microdiscectomy
D. Laminectomy and fusion
E. None of the above

Compression of nerve roots in the
lateral recess (lateral recess syndrome) can occur between
a hypertrophied superior articular facet (dorsally), the
pedicle (laterally), and the inferior vertebral body (ventrally). Medially, the lateral recess opens toward the spinal
canal/thecal sac. The characteristic feature of lateral recess
syndrome is that of radicular symptoms that occur mainly
when the patient is walking or standing and are relieved
by Sitting, squatting forward, lying on either side, andlor
postures that accentuate lumbar kyphosis. This is opposite
to what is seen with patients harboring discogenic disease ,
who are uncomfortable while sitting. With the lateral recess
syndrome, adequate decompression involves laminectomy
with resec tion of the medial third of the hypertrophied facet
(media l facetectomy), which is usually the superior articular
process (Wi ll in s, pp. 3841- 3845).

48
Q

Match each of the following procedures with the
potential complication using each answer once, more than
once, or not at all.

Dysartbria and cognitive decline
A. Cordotomy
B. Periaqueductal gray stimulation
C. Percutaneous trigeminal electrocautery
D. Sympathectomy
E. Bilateral thalamotomy
F. Pallidotomy
G. ComIllisuralmyelotomy

A

A. Cordotomy
B. Periaqueductal gray stimulation
C. Percutaneous trigeminal electrocautery
D. Sympathectomy
**E. Bilateral thalamotomy **
F. Pallidotomy
G. ComIllisuralmyelotomy

Direct sectioning of
the spinothalamiC tract (cordotomy) is very effective for
unilateral pain below the upper chest region, however, it is
associated with many complications and is usually performed
only in terminal patients. Complications of cordotomy include hemiparesis, respiratory depression (Ondine’s curse with
bilateral procedures), and dysesthesias. Midline myelotomies
can also be performed to interrupt the decussating fibers
of the spinothalamic tract. This can be quite effective in
the treatment of chronic pelvic pain secondary to cancer
but is associated with leg wealmess, dysesthesias, and
bladder dysfunction. Chronic deep brain stimulation of the
VPL and VPM nuclei of the thalamus as well as the periaqueductal gray (PAG) has been performed in the treatment of
thalamic pain states, postherpetic neuralgia, and causalgia.
Stimulation of the PAG has been associated with eye movement disorders, pupillary dilation, and the feeling of fear.
Complications of pallidotomy include injury to the adjacent
internal capsule (hemiparesis) and optic tract (homonymous
hemianopsia), while complications of bilateral thalamotomy
include speech problems and congnitive decline. Horner’s
syndrome, pneumothorax, intercostal neuralgias, and spinal
cord injury can occur after sympathectomy, while anesthesia dolorosa has been reported to occur after percutaneous
trigeminal electrocautery (Kaye and Blacl~, pp. 1431-1445,
1481; Greenberg, pp. 352- 354, 361, 365-370, 373-380;
Youmans, pp.3025-3030,3045- 3048,3067- 3070,3101-
3102; Will~ins, pp. 4055-4059).

49
Q

Match each of the following procedures with the
potential complication using each answer once, more than
once, or not at all.

Hemiparesis, homonymous hemi anopia
A. Cordotomy
B. Periaqueductal gray stimulation
C. Percutaneous trigeminal electrocautery
D. Sympathectomy
E. Bilateral thalamotomy
F. Pallidotomy
G. ComIllisuralmyelotomy

A

A. Cordotomy
B. Periaqueductal gray stimulation
C. Percutaneous trigeminal electrocautery
D. Sympathectomy
E. Bilateral thalamotomy
**F. Pallidotomy **
G. ComIllisuralmyelotomy

Direct sectioning of
the spinothalamiC tract (cordotomy) is very effective for
unilateral pain below the upper chest region, however, it is
associated with many complications and is usually performed
only in terminal patients. Complications of cordotomy include hemiparesis, respiratory depression (Ondine’s curse with
bilateral procedures), and dysesthesias. Midline myelotomies
can also be performed to interrupt the decussating fibers
of the spinothalamic tract. This can be quite effective in
the treatment of chronic pelvic pain secondary to cancer
but is associated with leg wealmess, dysesthesias, and
bladder dysfunction. Chronic deep brain stimulation of the
VPL and VPM nuclei of the thalamus as well as the periaqueductal gray (PAG) has been performed in the treatment of
thalamic pain states, postherpetic neuralgia, and causalgia.
Stimulation of the PAG has been associated with eye movement disorders, pupillary dilation, and the feeling of fear.
Complications of pallidotomy include injury to the adjacent
internal capsule (hemiparesis) and optic tract (homonymous
hemianopsia), while complications of bilateral thalamotomy
include speech problems and congnitive decline. Horner’s
syndrome, pneumothorax, intercostal neuralgias, and spinal
cord injury can occur after sympathectomy, while anesthesia dolorosa has been reported to occur after percutaneous
trigeminal electrocautery (Kaye and Blacl~, pp. 1431-1445,
1481; Greenberg, pp. 352- 354, 361, 365-370, 373-380;
Youmans, pp.3025-3030,3045- 3048,3067- 3070,3101-
3102; Will~ins, pp. 4055-4059).

50
Q

Match each of the following procedures with the
potential complication using each answer once, more than
once, or not at all.

“Ondine’s curse”
A. Cordotomy
B. Periaqueductal gray stimulation
C. Percutaneous trigeminal electrocautery
D. Sympathectomy
E. Bilateral thalamotomy
F. Pallidotomy
G. ComIllisuralmyelotomy

A

**A. Cordotomy **
B. Periaqueductal gray stimulation
C. Percutaneous trigeminal electrocautery
D. Sympathectomy
E. Bilateral thalamotomy
F. Pallidotomy
G. ComIllisuralmyelotomy

Direct sectioning of
the spinothalamiC tract (cordotomy) is very effective for
unilateral pain below the upper chest region, however, it is
associated with many complications and is usually performed
only in terminal patients. Complications of cordotomy include hemiparesis, respiratory depression (Ondine’s curse with
bilateral procedures), and dysesthesias. Midline myelotomies
can also be performed to interrupt the decussating fibers
of the spinothalamic tract. This can be quite effective in
the treatment of chronic pelvic pain secondary to cancer
but is associated with leg wealmess, dysesthesias, and
bladder dysfunction. Chronic deep brain stimulation of the
VPL and VPM nuclei of the thalamus as well as the periaqueductal gray (PAG) has been performed in the treatment of
thalamic pain states, postherpetic neuralgia, and causalgia.
Stimulation of the PAG has been associated with eye movement disorders, pupillary dilation, and the feeling of fear.
Complications of pallidotomy include injury to the adjacent
internal capsule (hemiparesis) and optic tract (homonymous
hemianopsia), while complications of bilateral thalamotomy
include speech problems and congnitive decline. Horner’s
syndrome, pneumothorax, intercostal neuralgias, and spinal
cord injury can occur after sympathectomy, while anesthesia dolorosa has been reported to occur after percutaneous
trigeminal electrocautery (Kaye and Blacl~, pp. 1431-1445,
1481; Greenberg, pp. 352- 354, 361, 365-370, 373-380;
Youmans, pp.3025-3030,3045- 3048,3067- 3070,3101-
3102; Will~ins, pp. 4055-4059).

51
Q

Match each of the following procedures with the
potential complication using each answer once, more than
once, or not at all.

Eye movement disorder, pupillary dilation, feeling of fear
A. Cordotomy
B. Periaqueductal gray stimulation
C. Percutaneous trigeminal electrocautery
D. Sympathectomy
E. Bilateral thalamotomy
F. Pallidotomy
G. ComIllisuralmyelotomy

A

A. Cordotomy
**B. Periaqueductal gray stimulation **
C. Percutaneous trigeminal electrocautery
D. Sympathectomy
E. Bilateral thalamotomy
F. Pallidotomy
G. ComIllisuralmyelotomy

Direct sectioning of
the spinothalamiC tract (cordotomy) is very effective for
unilateral pain below the upper chest region, however, it is
associated with many complications and is usually performed
only in terminal patients. Complications of cordotomy include hemiparesis, respiratory depression (Ondine’s curse with
bilateral procedures), and dysesthesias. Midline myelotomies
can also be performed to interrupt the decussating fibers
of the spinothalamic tract. This can be quite effective in
the treatment of chronic pelvic pain secondary to cancer
but is associated with leg wealmess, dysesthesias, and
bladder dysfunction. Chronic deep brain stimulation of the
VPL and VPM nuclei of the thalamus as well as the periaqueductal gray (PAG) has been performed in the treatment of
thalamic pain states, postherpetic neuralgia, and causalgia.
Stimulation of the PAG has been associated with eye movement disorders, pupillary dilation, and the feeling of fear.
Complications of pallidotomy include injury to the adjacent
internal capsule (hemiparesis) and optic tract (homonymous
hemianopsia), while complications of bilateral thalamotomy
include speech problems and congnitive decline. Horner’s
syndrome, pneumothorax, intercostal neuralgias, and spinal
cord injury can occur after sympathectomy, while anesthesia dolorosa has been reported to occur after percutaneous
trigeminal electrocautery (Kaye and Blacl~, pp. 1431-1445,
1481; Greenberg, pp. 352- 354, 361, 365-370, 373-380;
Youmans, pp.3025-3030,3045- 3048,3067- 3070,3101-
3102; Will~ins, pp. 4055-4059).

52
Q

Match each of the following procedures with the
potential complication using each answer once, more than
once, or not at all.

Horner’s syndrome
A. Cordotomy
B. Periaqueductal gray stimulation
C. Percutaneous trigeminal electrocautery
D. Sympathectomy
E. Bilateral thalamotomy
F. Pallidotomy
G. ComIllisuralmyelotomy

A

A. Cordotomy
B. Periaqueductal gray stimulation
C. Percutaneous trigeminal electrocautery
**D. Sympathectomy **
E. Bilateral thalamotomy
F. Pallidotomy
G. ComIllisuralmyelotomy

Direct sectioning of
the spinothalamiC tract (cordotomy) is very effective for
unilateral pain below the upper chest region, however, it is
associated with many complications and is usually performed
only in terminal patients. Complications of cordotomy include hemiparesis, respiratory depression (Ondine’s curse with
bilateral procedures), and dysesthesias. Midline myelotomies
can also be performed to interrupt the decussating fibers
of the spinothalamic tract. This can be quite effective in
the treatment of chronic pelvic pain secondary to cancer
but is associated with leg wealmess, dysesthesias, and
bladder dysfunction. Chronic deep brain stimulation of the
VPL and VPM nuclei of the thalamus as well as the periaqueductal gray (PAG) has been performed in the treatment of
thalamic pain states, postherpetic neuralgia, and causalgia.
Stimulation of the PAG has been associated with eye movement disorders, pupillary dilation, and the feeling of fear.
Complications of pallidotomy include injury to the adjacent
internal capsule (hemiparesis) and optic tract (homonymous
hemianopsia), while complications of bilateral thalamotomy
include speech problems and congnitive decline. Horner’s
syndrome, pneumothorax, intercostal neuralgias, and spinal
cord injury can occur after sympathectomy, while anesthesia dolorosa has been reported to occur after percutaneous
trigeminal electrocautery (Kaye and Blacl~, pp. 1431-1445,
1481; Greenberg, pp. 352- 354, 361, 365-370, 373-380;
Youmans, pp.3025-3030,3045- 3048,3067- 3070,3101-
3102; Will~ins, pp. 4055-4059).

53
Q

Match each of the following procedures with the
potential complication using each answer once, more than
once, or not at all.

\nesthesia dolorosa
A. Cordotomy
B. Periaqueductal gray stimulation
C. Percutaneous trigeminal electrocautery
D. Sympathectomy
E. Bilateral thalamotomy
F. Pallidotomy
G. ComIllisuralmyelotomy

A

A. Cordotomy
B. Periaqueductal gray stimulation
**C. Percutaneous trigeminal electrocautery **
D. Sympathectomy
E. Bilateral thalamotomy
F. Pallidotomy
G. ComIllisuralmyelotomy

Direct sectioning of
the spinothalamiC tract (cordotomy) is very effective for
unilateral pain below the upper chest region, however, it is
associated with many complications and is usually performed
only in terminal patients. Complications of cordotomy include hemiparesis, respiratory depression (Ondine’s curse with
bilateral procedures), and dysesthesias. Midline myelotomies
can also be performed to interrupt the decussating fibers
of the spinothalamic tract. This can be quite effective in
the treatment of chronic pelvic pain secondary to cancer
but is associated with leg wealmess, dysesthesias, and
bladder dysfunction. Chronic deep brain stimulation of the
VPL and VPM nuclei of the thalamus as well as the periaqueductal gray (PAG) has been performed in the treatment of
thalamic pain states, postherpetic neuralgia, and causalgia.
Stimulation of the PAG has been associated with eye movement disorders, pupillary dilation, and the feeling of fear.
Complications of pallidotomy include injury to the adjacent
internal capsule (hemiparesis) and optic tract (homonymous
hemianopsia), while complications of bilateral thalamotomy
include speech problems and congnitive decline. Horner’s
syndrome, pneumothorax, intercostal neuralgias, and spinal
cord injury can occur after sympathectomy, while anesthesia dolorosa has been reported to occur after percutaneous
trigeminal electrocautery (Kaye and Blacl~, pp. 1431-1445,
1481; Greenberg, pp. 352- 354, 361, 365-370, 373-380;
Youmans, pp.3025-3030,3045- 3048,3067- 3070,3101-
3102; Will~ins, pp. 4055-4059).

54
Q

Match each of the following procedures with the
potential complication using each answer once, more than
once, or not at all.

Leg wealmess, dysesthesias, bladder dysfunction
A. Cordotomy
B. Periaqueductal gray stimulation
C. Percutaneous trigeminal electrocautery
D. Sympathectomy
E. Bilateral thalamotomy
F. Pallidotomy
G. ComIllisuralmyelotomy

A

A. Cordotomy
B. Periaqueductal gray stimulation
C. Percutaneous trigeminal electrocautery
D. Sympathectomy
E. Bilateral thalamotomy
F. Pallidotomy
**G. ComIllisuralmyelotomy **

Direct sectioning of
the spinothalamiC tract (cordotomy) is very effective for
unilateral pain below the upper chest region, however, it is
associated with many complications and is usually performed
only in terminal patients. Complications of cordotomy include hemiparesis, respiratory depression (Ondine’s curse with
bilateral procedures), and dysesthesias. Midline myelotomies
can also be performed to interrupt the decussating fibers
of the spinothalamic tract. This can be quite effective in
the treatment of chronic pelvic pain secondary to cancer
but is associated with leg wealmess, dysesthesias, and
bladder dysfunction. Chronic deep brain stimulation of the
VPL and VPM nuclei of the thalamus as well as the periaqueductal gray (PAG) has been performed in the treatment of
thalamic pain states, postherpetic neuralgia, and causalgia.
Stimulation of the PAG has been associated with eye movement disorders, pupillary dilation, and the feeling of fear.
Complications of pallidotomy include injury to the adjacent
internal capsule (hemiparesis) and optic tract (homonymous
hemianopsia), while complications of bilateral thalamotomy
include speech problems and congnitive decline. Horner’s
syndrome, pneumothorax, intercostal neuralgias, and spinal
cord injury can occur after sympathectomy, while anesthesia dolorosa has been reported to occur after percutaneous
trigeminal electrocautery (Kaye and Blacl~, pp. 1431-1445,
1481; Greenberg, pp. 352- 354, 361, 365-370, 373-380;
Youmans, pp.3025-3030,3045- 3048,3067- 3070,3101-
3102; Will~ins, pp. 4055-4059).

55
Q

All of the following are established procedures for the
treatment of trigeminal neuralgia EXCEPT?
A. Glycerol rhizolysis
B. Balloon decompression
C. Radiofrequency thermocoagulation
D. i’l’licrovascular decompression
E. Peripheral alcohol injection

A

A. Glycerol rhizolysis
**B. Balloon decompression **
C. Radiofrequency thermocoagulation
D. i’l’licrovascular decompression
E. Peripheral alcohol injection

Peripheral alcohol injection, glycerol rhizolysis, radiofrequency thermocoagulation, and microvascular decompression are all established procedures for the treatment
of trigeminal neuralgia. Peripheral balloon compression
instead of decompression is a modification of the observation
that open surgical decompression of the ganglion could lead
to significant pain relief in trigeminal neuralgia (Kaye and
Blacl~, pp. 1616-1633; Greenberg, pp. 373-380).

56
Q

A 58-year-old male with rheumatoid arthritis presents to the emergency department with intolerable neck
pain and cervical myelopathy. On MRI, he is found to have
superior migration of the odontoid (SMO) process through
the foramen magnum (cranial settling) and compression of
the brainstem by the odontoid process itself

All of the following information is important to gather
preoperatively in patients with craniocervical junction
(CCJ) abnormalities EXCEPT?

A. The evaluation of craniocervical stability
B. EMG and nerve conduction studies (NCS) to identify
the extent of peripheral nerve damage
C. Whether there is an associated syrinx
D. The extent of ventral compression
E. Presence of abnormal ossification centers and epipbyseal growth plates in children, as this may alter
trea tment strategies

A

A. The evaluation of craniocervical stability
**B. EMG and nerve conduction studies (NCS) to identify
the extent of peripheral nerve damage **
C. Whether there is an associated syrinx
D. The extent of ventral compression
E. Presence of abnormal ossification centers and epipbyseal growth plates in children, as this may alter
trea tment strategies

Craniocervical
junction (CCJ) abnormalities can often be very difficult
to manage, with the primary goal being to relieve the
compression at the cervicomedullary junction. They are
commonly seen in patients with Chiari malformation or
rheumatoid arthritis. With reducible lesions, stabilization
is essential to maintain neural decompression, while for
irreducible lesions, decompression at the site of encroachment (ventral or posterior) as well as stabilization are often
required.
Patients with rheumatoid arthritis are at risk for developing atlantoaxial instability (AAI); superior migration of the
odontoid process (SMO), also lmown as cranial settling; and
subaxial subluxations (SAS). For rheumatoid patients with
reducible leSions, immobilization alone with posterior spinal
or craniospinal fusion without decompressive procedures is
the mainstay of treatment.
Late-onset deterioration in patients with rheumatoid
arthritis or Chiari malformations in the pattern seen in this
patient is concerning for syritL: or syringomyelia formation
(Kaye and Blacl~, pp. 1755-1770; Will~ins, pp. 3789- 3790;
Youmans, pp. 4569- 4580).

57
Q

A 58-year-old male with rheumatoid arthritis presents to the emergency department with intolerable neck
pain and cervical myelopathy. On MRI, he is found to have
superior migration of the odontoid (SMO) process through
the foramen magnum (cranial settling) and compression of
the brainstem by the odontoid process itself

Dynamic imaging studies of the craniocervical junction
reveal instability. The neurosurgeon elects to employ gentle
cervical traction for 3 days with good success in reducing the
abnormality. After 3 days of traction, the patient’s neck pain
significantly improves, and i’vIRI reveals minimal brainstem
compression in the reduced position. What should be the
next course of management?

A. Posterior cervical laminectomy
B. Posterior cervical laminectomy, suboccipital craniectomy, and fusion
C. Cervical traction for another week to attempt to further reduce the abnormality before embarking on any
surgical procedure
D. Immobilization alone with posterior cervical fusion
without a decompression
E. Transoral odontectomy followed by posterior cervical
decompression, suboccipital craniectomy, and fusion

A

A. Posterior cervical laminectomy
B. Posterior cervical laminectomy, suboccipital craniectomy, and fusion
C. Cervical traction for another week to attempt to further reduce the abnormality before embarking on any
surgical procedure
**D. Immobilization alone with posterior cervical fusion
without a decompression **
E. Transoral odontectomy followed by posterior cervical
decompression, suboccipital craniectomy, and fusion

Craniocervical
junction (CCJ) abnormalities can often be very difficult
to manage, with the primary goal being to relieve the
compression at the cervicomedullary junction. They are
commonly seen in patients with Chiari malformation or
rheumatoid arthritis. With reducible lesions, stabilization
is essential to maintain neural decompression, while for
irreducible lesions, decompression at the site of encroachment (ventral or posterior) as well as stabilization are often
required.
Patients with rheumatoid arthritis are at risk for developing atlantoaxial instability (AAI); superior migration of the
odontoid process (SMO), also lmown as cranial settling; and
subaxial subluxations (SAS). For rheumatoid patients with
reducible leSions, immobilization alone with posterior spinal
or craniospinal fusion without decompressive procedures is
the mainstay of treatment.
Late-onset deterioration in patients with rheumatoid
arthritis or Chiari malformations in the pattern seen in this
patient is concerning for syritL: or syringomyelia formation
(Kaye and Blacl~, pp. 1755-1770; Will~ins, pp. 3789- 3790;
Youmans, pp. 4569- 4580).

58
Q

A 58-year-old male with rheumatoid arthritis presents to the emergency department with intolerable neck
pain and cervical myelopathy. On MRI, he is found to have
superior migration of the odontoid (SMO) process through
the foramen magnum (cranial settling) and compression of
the brainstem by the odontoid process itself

One year later the patient experiences progressive
wealmess in his legs, ataxia, and bladder incontinence. His
strength in the upper extremities is preserved, and he has no
evidence of cranial nerve abnormalities. Plain films and CT
scan of the craniocervical junction are unremarkable. What
should be the next diagnostic test employed?

A. CT of the brain to look for hydrocephalus
B. EMG and NCS to identify the extent of peripheral
nerve damage
C. Screening MRI of the spine
D. Bladder urodynamic testing
E. Dynamic films of the cervical spine to evaluate for
pseudoarthrosis and instability

A

A. CT of the brain to look for hydrocephalus
B. EMG and NCS to identify the extent of peripheral
nerve damage
**C. Screening MRI of the spine **
D. Bladder urodynamic testing
E. Dynamic films of the cervical spine to evaluate for
pseudoarthrosis and instability

Craniocervical
junction (CCJ) abnormalities can often be very difficult
to manage, with the primary goal being to relieve the
compression at the cervicomedullary junction. They are
commonly seen in patients with Chiari malformation or
rheumatoid arthritis. With reducible lesions, stabilization
is essential to maintain neural decompression, while for
irreducible lesions, decompression at the site of encroachment (ventral or posterior) as well as stabilization are often
required.
Patients with rheumatoid arthritis are at risk for developing atlantoaxial instability (AAI); superior migration of the
odontoid process (SMO), also lmown as cranial settling; and
subaxial subluxations (SAS). For rheumatoid patients with
reducible leSions, immobilization alone with posterior spinal
or craniospinal fusion without decompressive procedures is
the mainstay of treatment.
Late-onset deterioration in patients with rheumatoid
arthritis or Chiari malformations in the pattern seen in this
patient is concerning for syritL: or syringomyelia formation
(Kaye and Blacl~, pp. 1755-1770; Will~ins, pp. 3789- 3790;
Youmans, pp. 4569- 4580).

59
Q

A surgeon utilizes an infratemporal fossa approach to
remove a large infiltrating tumor of the cranial base. He
comes across the shaded structure depicted by the arrow
below (Figure 6.59-6.61Q). How many muscles attach to
this structure?
A. 2
B. 3
C. 4
D. 5
E. 6

A

A. 2
**B. 3 **
C. 4
D. 5
E. 6

The stylOid process gives rise to the stylohyoid (VII), styloglossus (XII), and stylopharyngeal muscles
(IX) of the visceral neck as well as the stylomandibular and
stylohyoid ligaments. It is a remnant of the second brachial
arch (Youmans, p. 36).

60
Q

A surgeon utilizes an infratemporal fossa approach to
remove a large infiltrating tumor of the cranial base. He
comes across the shaded structure depicted by the arrow
below. Which cranial nerves innervate these muscles?
A. VII, IX
B. VII, IX, XII
C. IX, X, XII
D. V, VII, IX
E. X, XII

A

A. VII, IX
**B. VII, IX, XII **
C. IX, X, XII
D. V, VII, IX
E. X, XII

The stylOid process gives rise to the stylohyoid (VII), styloglossus (XII), and stylopharyngeal muscles
(IX) of the visceral neck as well as the stylomandibular and
stylohyoid ligaments. It is a remnant of the second brachial
arch (Youmans, p. 36).

61
Q

A surgeon utilizes an infratemporal fossa approach to
remove a large infiltrating tumor of the cranial base. He
comes across the shaded structure depicted by the arrow
below

How many ligaments attach to this structure?
A. 1
B. 2
C. 3
D. 4
E. 5

A

A. 1
B. 2
**C. 3 **
D. 4
E. 5

The stylOid process gives rise to the stylohyoid (VII), styloglossus (XII), and stylopharyngeal muscles
(IX) of the visceral neck as well as the stylomandibular and
stylohyoid ligaments. It is a remnant of the second brachial
arch (Youmans, p. 36).

62
Q

Match each of the following questions with
the most likely fracture pattern (letterhead) depicted in
picture bellow

Most likely to cause wealmess of the extensor muscles
of the wrist and hand; extension of forearm typically not
affected; sensation of dorsal hand affected

A

B

Fracture of the
proximal humerus (A) can result in injury to the axillary
nerve (C5-6), which innervates the teres minor and deltoid
muscles. This can result in sensory loss at the shoulder as
well as shoulder abduction wealmess. There is also a chance
of concomitant brachial plexus injury with such a fracture
due to the proximity of the proximal humerus to the brachial
plexus. The radial nerve runs down the posterior aspect
of the arm and is at risk for injury during fractures of the
midhumeral shaft as it winds around the spiral groove (8).
This could cause paralysis of the wrist and hand extensor
muscles. Since the fibers that innervate the triceps muscle
often arise proximal to the spiral groove, extension of the
forearm may not be affected by midhumeral fractures, and
some supination is possible due to an intact biceps brachii
muscle. Fracture in the vicinity of the medial epicondyle
may result in ulnar nerve damage only (C), which can produce wealmess of flexion and adduction of wrist, pa ralysis
of hypothenar muscles and most deep muscles of the hand,
as well as some weakness in select thenar muscles. Injury of
the ulnar nerve by fracture of the distal ulna can result in
wealmess or paralysis of hypothenar, some thenar, and
intrinsic hand muscles but often spares innervation of the
wrist (flexion, adduction), since these nerves often arise
more proximally. If there is also a concomitant distal radial
fracture, injury to the median nerve may accompany the
ulnar nerve injury (F) and produce loss of sensation of the
lateral side of the palm without sensory loss on the palmar
sides of the first, second, and third digits (superficial branch
of the median nerve) as well as marked weakness of thumb

63
Q

Match each of the following questions with
the most likely fracture pattern (letterhead) depicted in
picture bellow

May result in teres minor wealmess

A

A

Fracture of the
proximal humerus (A) can result in injury to the axillary
nerve (C5-6), which innervates the teres minor and deltoid
muscles. This can result in sensory loss at the shoulder as
well as shoulder abduction wealmess. There is also a chance
of concomitant brachial plexus injury with such a fracture
due to the proximity of the proximal humerus to the brachial
plexus. The radial nerve runs down the posterior aspect
of the arm and is at risk for injury during fractures of the
midhumeral shaft as it winds around the spiral groove (8).
This could cause paralysis of the wrist and hand extensor
muscles. Since the fibers that innervate the triceps muscle
often arise proximal to the spiral groove, extension of the
forearm may not be affected by midhumeral fractures, and
some supination is possible due to an intact biceps brachii
muscle. Fracture in the vicinity of the medial epicondyle
may result in ulnar nerve damage only (C), which can produce wealmess of flexion and adduction of wrist, pa ralysis
of hypothenar muscles and most deep muscles of the hand,
as well as some weakness in select thenar muscles. Injury of
the ulnar nerve by fracture of the distal ulna can result in
wealmess or paralysis of hypothenar, some thenar, and
intrinsic hand muscles but often spares innervation of the
wrist (flexion, adduction), since these nerves often arise
more proximally. If there is also a concomitant distal radial
fracture, injury to the median nerve may accompany the
ulnar nerve injury (F) and produce loss of sensation of the
lateral side of the palm without sensory loss on the palmar
sides of the first, second, and third digits (superficial branch
of the median nerve) as well as marked weakness of thumb

64
Q

Match each of the following questions with
the most likely fracture pattern (letterhead) depicted in
picture bellow

Weakness of flexion and adduction of wrist, paralysis of
hypothenar muscles and most deep muscles of the hand,
some wealmess in thenar muscles

A

C

Fracture of the
proximal humerus (A) can result in injury to the axillary
nerve (C5-6), which innervates the teres minor and deltoid
muscles. This can result in sensory loss at the shoulder as
well as shoulder abduction wealmess. There is also a chance
of concomitant brachial plexus injury with such a fracture
due to the proximity of the proximal humerus to the brachial
plexus. The radial nerve runs down the posterior aspect
of the arm and is at risk for injury during fractures of the
midhumeral shaft as it winds around the spiral groove (8).
This could cause paralysis of the wrist and hand extensor
muscles. Since the fibers that innervate the triceps muscle
often arise proximal to the spiral groove, extension of the
forearm may not be affected by midhumeral fractures, and
some supination is possible due to an intact biceps brachii
muscle. Fracture in the vicinity of the medial epicondyle
may result in ulnar nerve damage only (C), which can produce wealmess of flexion and adduction of wrist, pa ralysis
of hypothenar muscles and most deep muscles of the hand,
as well as some weakness in select thenar muscles. Injury of
the ulnar nerve by fracture of the distal ulna can result in
wealmess or paralysis of hypothenar, some thenar, and
intrinsic hand muscles but often spares innervation of the
wrist (flexion, adduction), since these nerves often arise
more proximally. If there is also a concomitant distal radial
fracture, injury to the median nerve may accompany the
ulnar nerve injury (F) and produce loss of sensation of the
lateral side of the palm without sensory loss on the palmar
sides of the first, second, and third digits (superficial branch
of the median nerve) as well as marked weakness of thumb

65
Q

Match each of the following questions with
the most likely fracture pattern (letterhead) depicted in
picture bellow

Shoulder abduction wealmess

A

A

Fracture of the
proximal humerus (A) can result in injury to the axillary
nerve (C5-6), which innervates the teres minor and deltoid
muscles. This can result in sensory loss at the shoulder as
well as shoulder abduction wealmess. There is also a chance
of concomitant brachial plexus injury with such a fracture
due to the proximity of the proximal humerus to the brachial
plexus. The radial nerve runs down the posterior aspect
of the arm and is at risk for injury during fractures of the
midhumeral shaft as it winds around the spiral groove (8).
This could cause paralysis of the wrist and hand extensor
muscles. Since the fibers that innervate the triceps muscle
often arise proximal to the spiral groove, extension of the
forearm may not be affected by midhumeral fractures, and
some supination is possible due to an intact biceps brachii
muscle. Fracture in the vicinity of the medial epicondyle
may result in ulnar nerve damage only (C), which can produce wealmess of flexion and adduction of wrist, pa ralysis
of hypothenar muscles and most deep muscles of the hand,
as well as some weakness in select thenar muscles. Injury of
the ulnar nerve by fracture of the distal ulna can result in
wealmess or paralysis of hypothenar, some thenar, and
intrinsic hand muscles but often spares innervation of the
wrist (flexion, adduction), since these nerves often arise
more proximally. If there is also a concomitant distal radial
fracture, injury to the median nerve may accompany the
ulnar nerve injury (F) and produce loss of sensation of the
lateral side of the palm without sensory loss on the palmar
sides of the first, second, and third digits (superficial branch
of the median nerve) as well as marked weakness of thumb

66
Q

Match each of the following questions with
the most likely fracture pattern (letterhead) depicted in
picture bellow

High likelihood of ulnar nerve injury only

A

C

Fracture of the
proximal humerus (A) can result in injury to the axillary
nerve (C5-6), which innervates the teres minor and deltoid
muscles. This can result in sensory loss at the shoulder as
well as shoulder abduction wealmess. There is also a chance
of concomitant brachial plexus injury with such a fracture
due to the proximity of the proximal humerus to the brachial
plexus. The radial nerve runs down the posterior aspect
of the arm and is at risk for injury during fractures of the
midhumeral shaft as it winds around the spiral groove (8).
This could cause paralysis of the wrist and hand extensor
muscles. Since the fibers that innervate the triceps muscle
often arise proximal to the spiral groove, extension of the
forearm may not be affected by midhumeral fractures, and
some supination is possible due to an intact biceps brachii
muscle. Fracture in the vicinity of the medial epicondyle
may result in ulnar nerve damage only (C), which can produce wealmess of flexion and adduction of wrist, pa ralysis
of hypothenar muscles and most deep muscles of the hand,
as well as some weakness in select thenar muscles. Injury of
the ulnar nerve by fracture of the distal ulna can result in
wealmess or paralysis of hypothenar, some thenar, and
intrinsic hand muscles but often spares innervation of the
wrist (flexion, adduction), since these nerves often arise
more proximally. If there is also a concomitant distal radial
fracture, injury to the median nerve may accompany the
ulnar nerve injury (F) and produce loss of sensation of the
lateral side of the palm without sensory loss on the palmar
sides of the first, second, and third digits (superficial branch
of the median nerve) as well as marked weakness of thumb

67
Q

Match each of the following questions with
the most likely fracture pattern (letterhead) depicted in
picture bellow

Median l1e’rve damage, paralysis of hypothenar muscles,
some thenar muscles, and most of the deep muscles of the
hand; flexion and adduction of wrist spared

A

F

Fracture of the
proximal humerus (A) can result in injury to the axillary
nerve (C5-6), which innervates the teres minor and deltoid
muscles. This can result in sensory loss at the shoulder as
well as shoulder abduction wealmess. There is also a chance
of concomitant brachial plexus injury with such a fracture
due to the proximity of the proximal humerus to the brachial
plexus. The radial nerve runs down the posterior aspect
of the arm and is at risk for injury during fractures of the
midhumeral shaft as it winds around the spiral groove (8).
This could cause paralysis of the wrist and hand extensor
muscles. Since the fibers that innervate the triceps muscle
often arise proximal to the spiral groove, extension of the
forearm may not be affected by midhumeral fractures, and
some supination is possible due to an intact biceps brachii
muscle. Fracture in the vicinity of the medial epicondyle
may result in ulnar nerve damage only (C), which can produce wealmess of flexion and adduction of wrist, pa ralysis
of hypothenar muscles and most deep muscles of the hand,
as well as some weakness in select thenar muscles. Injury of
the ulnar nerve by fracture of the distal ulna can result in
wealmess or paralysis of hypothenar, some thenar, and
intrinsic hand muscles but often spares innervation of the
wrist (flexion, adduction), since these nerves often arise
more proximally. If there is also a concomitant distal radial
fracture, injury to the median nerve may accompany the
ulnar nerve injury (F) and produce loss of sensation of the
lateral side of the palm without sensory loss on the palmar
sides of the first, second, and third digits (superficial branch
of the median nerve) as well as marked weakness of thumb

68
Q

Match each of the following questions with
the most likely fracture pattern (letterhead) depicted in
picture bellow

Can be associated with brachial plexus injuries

A

A

Fracture of the
proximal humerus (A) can result in injury to the axillary
nerve (C5-6), which innervates the teres minor and deltoid
muscles. This can result in sensory loss at the shoulder as
well as shoulder abduction wealmess. There is also a chance
of concomitant brachial plexus injury with such a fracture
due to the proximity of the proximal humerus to the brachial
plexus. The radial nerve runs down the posterior aspect
of the arm and is at risk for injury during fractures of the
midhumeral shaft as it winds around the spiral groove (8).
This could cause paralysis of the wrist and hand extensor
muscles. Since the fibers that innervate the triceps muscle
often arise proximal to the spiral groove, extension of the
forearm may not be affected by midhumeral fractures, and
some supination is possible due to an intact biceps brachii
muscle. Fracture in the vicinity of the medial epicondyle
may result in ulnar nerve damage only (C), which can produce wealmess of flexion and adduction of wrist, pa ralysis
of hypothenar muscles and most deep muscles of the hand,
as well as some weakness in select thenar muscles. Injury of
the ulnar nerve by fracture of the distal ulna can result in
wealmess or paralysis of hypothenar, some thenar, and
intrinsic hand muscles but often spares innervation of the
wrist (flexion, adduction), since these nerves often arise
more proximally. If there is also a concomitant distal radial
fracture, injury to the median nerve may accompany the
ulnar nerve injury (F) and produce loss of sensation of the
lateral side of the palm without sensory loss on the palmar
sides of the first, second, and third digits (superficial branch
of the median nerve) as well as marked weakness of thumb

69
Q

Match each of the following questions with
the most likely fracture pattern (letterhead) depicted in
picture bellow

Most likely to cause combined radial, medial, and ulnar
nerve injuries

A

D

Fracture of the
proximal humerus (A) can result in injury to the axillary
nerve (C5-6), which innervates the teres minor and deltoid
muscles. This can result in sensory loss at the shoulder as
well as shoulder abduction wealmess. There is also a chance
of concomitant brachial plexus injury with such a fracture
due to the proximity of the proximal humerus to the brachial
plexus. The radial nerve runs down the posterior aspect
of the arm and is at risk for injury during fractures of the
midhumeral shaft as it winds around the spiral groove (8).
This could cause paralysis of the wrist and hand extensor
muscles. Since the fibers that innervate the triceps muscle
often arise proximal to the spiral groove, extension of the
forearm may not be affected by midhumeral fractures, and
some supination is possible due to an intact biceps brachii
muscle. Fracture in the vicinity of the medial epicondyle
may result in ulnar nerve damage only (C), which can produce wealmess of flexion and adduction of wrist, pa ralysis
of hypothenar muscles and most deep muscles of the hand,
as well as some weakness in select thenar muscles. Injury of
the ulnar nerve by fracture of the distal ulna can result in
wealmess or paralysis of hypothenar, some thenar, and
intrinsic hand muscles but often spares innervation of the
wrist (flexion, adduction), since these nerves often arise
more proximally. If there is also a concomitant distal radial
fracture, injury to the median nerve may accompany the
ulnar nerve injury (F) and produce loss of sensation of the
lateral side of the palm without sensory loss on the palmar
sides of the first, second, and third digits (superficial branch
of the median nerve) as well as marked weakness of thumb

70
Q

A 9-year-old girl presented to her pediatrician with
headaches and a bitemporal field cut. Her MRI is depicted
below (Figure 6.70Q). Which of the following would be true
regarding the endocrine outcome after surgical resec tion of
this tumor?
A. There is a 30% chance that she will develop diabetes
insipidus
B. The most serious and disabling probl em is the development of obesity, which occurs in about 50% of these
patients after surgery
C. Approximately 90% of patients will not require maintenance corticosteroid and thyroid replacement
therapy
D. Approximately 10% of patients will require growth
hormone replacement therapy
E. The endocrine outcome after surgery is very
unpredictable

A

A. There is a 30% chance that she will develop diabetes
insipidus
**B. The most serious and disabling probl em is the development of obesity, which occurs in about 50% of these
patients after surgery **
C. Approximately 90% of patients will not require maintenance corticosteroid and thyroid replacement
therapy
D. Approximately 10% of patients will require growth
hormone replacement therapy
E. The endocrine outcome after surgery is very
unpredictable

A significant number of children with craniopharyngiomas will have a significant endocrine abnormality after
surgery, which is quite predictable, The most serious complication appears to be obesity, which develops in about
50% of patients, These patients are unable to control their
appetite secondary to damage to the hypothalamic satiety
center. Growth hormone may benefit these patients, as it
appears to reduce body fat and increase lean body mass,
Nearly 50% of patients will require GIl-replacement therapy,
Diabetes insipidus occurs in about 90% of patients and
is often permanent. Moreover, about 90% of patients will
require hydrocortisone and thyroid replacement therapy
after surgery (Kaye and Blacl{, pp. 741- 748; Committee on
Education in Neurological Surgery, pp, 26, 116; Curtis et aI.,
pp.24- 27),

71
Q

A 42-year-old female was recently diagnosed with
spontaneous intracranial hypotension. All of the following
are frequently associated with this problem EXCEPT?
A. The headaches often resemble a post-lumbar puncture
headache
B. MRI scans with contrast may reveal enhancement of
the dura over the cerebral and cerebellar convexities
C. Spontaneous improvement is rarely seen, since CSF
leal{s are often identified adjacent to nerve roots
D. Spinal fluid may reveal elevated protein and pleocytosis
E. Analgesics containing caffeine may be helpful

A

A. The headaches often resemble a post-lumbar puncture
headache
B. MRI scans with contrast may reveal enhancement of
the dura over the cerebral and cerebellar convexities
**C. Spontaneous improvement is rarely seen, since CSF
leal{s are often identified adjacent to nerve roots **
D. Spinal fluid may reveal elevated protein and pleocytosis
E. Analgesics containing caffeine may be helpful

The headaches of spontaneous intracranial hypotension often resemble post-lumbar puncture headaches,
Ileadaches are usually worse in the upright position and are
generally relieved when the patient is lying down, The diagnosis is established by lumbar puncture, which reveals low
opening pressure or dry tap, It is not uncommon to have elevated protein and pleocytosis, It is postulated that this syndrome results from leakage of CSF to the outside neuraxis,
often around nerve roots, MR cisternography is often capable
of demonstrating the leak. Jugular compression will elevate
intracranial pressure but usually makes the headache worse ,
suggesting that low pressure may not be the only factor
responsible for the headaches, MRl scans often reveal dural
enhancement over the cerebral and cerebellar convexities,
tentorium, and fa lx, which usually resolves with resolution
of the symptoms, Treatment should be conservative, since
there is often spontaneous improvement. Analgesics containing caffeine and adequate hydration seem to help, In
some cases an epidural patch may be required; surgical closure of the fistula is rarely required (Committee on Education
in Neurological Surgery, pp. 17, 105; Kosmoslw, pp, 79- 83)

72
Q

All of the following lesions are appropriate for stereotactic
radiosurgery EXCEPT?
A. A 3-CI11’1 arteriovenous malformation in the brainstem
B. A l-cm right frontal and 2-cl11 left parietal metastatic
carcinoma from the lung
C. Recurrent glioblastoma of the left temporal lobe (2 C111J
)
D. A l-cm cavernoma of the right caudate nucleus that
previously hemorrhaged
E. Bilateral thalamic arteriovenous malformations (3 cmJ
)

A

A. A 3-CI11’1 arteriovenous malformation in the brainstem
B. A l-cm right frontal and 2-cl11 left parietal metastatic
carcinoma from the lung
C. Recurrent glioblastoma of the left temporal lobe (2 C111J
)
**D. A l-cm cavernoma of the right caudate nucleus that
previously hemorrhaged **
E. Bilateral thalamic arteriovenous malformations (3 cmJ
)

A long history of radiosurgical treatment for arteriovenous malformations exists, The best responses are often
obtained for lesions with volumes less than 4 cnl”, Multiple
metastatic lesions are also amenable to this treatment
modality, as are recurrent gliomas located in a variety of
locations including the brainstem, thalamus, or other eloquent areas, There is, however, controversy about the radiosurgical treatment of cavernomas, While disease control has been documented, some authors are concerned with the
potential of treatment-related complications such as recurrent hemorrhage after radiosurgery (Committee on Education
in Neurological Surgery, pp, 108-109; Friedman, pp, 832-
841; Lunsford, pp, 404-444; Blacl{, pp. 367S-369S)

73
Q

All of the following would reduce pain conduction or a
patient’s reaction to pain EXCEPT?
A. Stimulation of the periaqueductal gray
B. Prefrontal lobotomy
C. Cingulotomy
D. Hippocampectomy
E. Ventrolateral cordotomy

A

A. Stimulation of the periaqueductal gray
B. Prefrontal lobotomy
C. Cingulotomy
D. Hippocampectomy
E. Ventrolateral cordotomy

Prefrontal lobectomy, cingulotomy, ventrolateral
cordotomy, and periaqueductal gray stimulation may interrupt pain pathways or the response to painful stimuli,
Hippocampectomy does not interrupt these pathways but
may decrease the severity of complex partial seizures
(Youmans, pp. 3025- 3030; Greenberg, pp. 364-370),

74
Q

A 34-year-old female is involved in a motor vehicle collision, suffers a severe closed head injury (Figure 6. 74Q) , and
develops a significant posttraumatic tremor in the right arm.
Although posttraumatic tremors are generally difficult to
manage , which surgical procedure may help control tremors,
which are otherwise refractory to medical therapy?

A. Thalamic stimulation
B. Subthalamic nucleus stimulation
C . . Motor cortex stimulation
D. Capsulotomy
E. Multiple subpial transections

A

**A. Thalamic stimulation **
B. Subthalamic nucleus stimulation
C . . Motor cortex stimulation
D. Capsulotomy
E. Multiple subpial transections

Traumatic injury to the brainstem including the superior cerebellar peduncles and their connections can result
in severe tremor that may be delayed by weeks to months
following the brain injury, In some cases, improvement
or resolution occurs spontaneously, so some authors have
reconuuended a period of observation before considering
surgery, The largest published series for secondary tremors
is among patients with multiple sclerosiS, although a growing
body of information is now available for patients with posttraumatic tremors, “‘ith posttraumatic tremors, thalamic
stimulation (Vim) may be considered for those who are
refractory to medical management, although results have
been mL’(ed, Thalamotomy is another treatment option for
this group of patients, although postoperative dysarthria or
worsening pre-existing dysarthria is an especially troubling
complication in some of these studies (Tarsy, pp, 244- 254;
Youmans, pp. 2748-2750),

75
Q

A 36-year-old female has a complex aneurysm that
requires the use of cardiac arrest and profound hypothermia
during surgery. All of the following are potential physiologic
effects of profound hypothermia EXCEPT?

A. Increased blood viscosity
B. Hyperglycemia
C. Decreased corticosteroid release
D. Complement-mediated pneumonitis
E. Hypercoagulable state

A

A. Increased blood viscosity
B. Hyperglycemia
C. Decreased corticosteroid release
D. Complement-mediated pneumonitis
E. Hypercoagulable state

Profound hypothermia during Circulatory arrest can
result in various physiologic effects including increasing
blood viscosity, metabolic acidosis (underperfused tissue),
hyperglycemia (secondary to hypoinsulinemia), deCl’eased
corticosteroid secretion, complement-mediated pneumonitis,
renal failure (due to transient decrease in glomerular filtration rate, hemolysis, and blood product reactions), hepatic
failure , and hypothermia-induced coagulopathy (due to
platelet dysfunction and slowing of the enzymatic clotting
cascade) (Youmans, p. 1532),

76
Q

What is the most common physical manifesta tion of the
abnormality depicted by the angiogram below (Figure 6.76Q)?
A. Neckpain
B. Cervical bruit
C. Contralateral arm weakness or numbness
D. Dysesthesia
E. Transient vision losS

A

A. Neckpain
**B. Cervical bruit **
C. Contralateral arm weakness or numbness
D. Dysesthesia
E. Transient vision losS

The most cOl11mon ph);sical manifestation of extracranial carotid artery disease is a cervical bruit. The degree
of stenosis necessary to produce a bruit has been reported to
be as low as 25%, but in various studies its presence has been
found to indicate a significant level (> 50%) of stenosis on
angiography in at least 70% of patients, False-positive rates
of 10 to 40% and false-negative rates of 30 to 70% have been
reported for cervical bruits. The Framingham Study found
that the risk of stroke and TlAs in patients with bruits was
two to three times the risk for patients without bruits, Such
patients were also about 2,5 times more likely to have a heart
attack and l. 9 times more likely to die during the study
period (Youmans, p. 1622).

77
Q

What is the most likely
diagnosis ?
A. Echinococcus infection
B. Neurocysticercosis
C. Oryptococcus infection
D. Cytomegalovirus infection
E. Trichinosis

A

A. Echinococcus infection
**B. Neurocysticercosis **
C. Oryptococcus infection
D. Cytomegalovirus infection
E. Trichinosis

Neurocysticercosis (NCC) is the most common parasitic infection of the central nervous system (CNS)
worldwide. Humans are the definitive host for the adult tapeworm Taenia so/iwn, which thrives in the small intestine
without consequence. Fecal shedding of eggs usually leads to
ingestion of eggs in contaminated water or food by an intermediate host, typically humans or pig. Once inside the intestine, the eggs are released and produce primary larvae that
enter the Circulatory system. Hematogenous spread to muscular, ocular, and neural tissue then occurs. Once inside the
brain, the primary larvae develop into secondary larvae, the
cysticerci. Clinical manifestations of the neural form of
the disease are varied and nonspecific. This pleomorphism is
related to the number, size, and topography of the lesions.
Parenchymal disease (as in this case) is most common and
presents with seizures in 50 to 80% of patients. Treatment
typically includes antiepileptics, albendazole, or praziquantel , as well as a short course of steroids to reduce the
inflammatory reaction during antihelminthic treatment.
Niclosamide may be given orally to treat adult tapeworms
in the GI tract. Fluconazole is an antifungal and not used to
treat this disease process. The colonization of the ventricular
system often presents with rapid clinical deterioration due
to increased intracranial pressure from obstructive hydrocephalus. There is still controversy about the best treatment
for this form of the disease, but most authors advocate either
a trial of antihelminthic medication, endoscopic cyst resection, or microsurgery. Subarachnoid disease is usually more
difficult to manage because the cysts are usually multiple,
attain larger sizes, and produce severe basal meningitis, but
antihelminthic medications are typically first-line therapy
for this form of the disease. From the many tests performed,
current data indicate that enzyme-linked immunosorbent
assay (ELISA) and electroimmunotransfer blot (EITB) tests
are the most effective laboratory tests for diagnosis. Peripheral white blood cells, ova and parasites in stool, and eosinophilia are inconsistent and unreliable markers of disease.
This patient also harbored a fourth ventricular cyst and
obstructive hychocephalus, which accounts for the rounded
third ventricle (Greenberg, pp. 236- 239)

78
Q

This patient is most likely to present with ?
A. Headaches
B. Obtundation
C. Cranial nerve palsies
D. Fevers
E. Seizure

A

A. Headaches
B. Obtundation
C. Cranial nerve palsies
D. Fevers
**E. Seizure **

Neurocysticercosis (NCC) is the most common parasitic infection of the central nervous system (CNS)
worldwide. Humans are the definitive host for the adult tapeworm Taenia so/iwn, which thrives in the small intestine
without consequence. Fecal shedding of eggs usually leads to
ingestion of eggs in contaminated water or food by an intermediate host, typically humans or pig. Once inside the intestine, the eggs are released and produce primary larvae that
enter the Circulatory system. Hematogenous spread to muscular, ocular, and neural tissue then occurs. Once inside the
brain, the primary larvae develop into secondary larvae, the
cysticerci. Clinical manifestations of the neural form of
the disease are varied and nonspecific. This pleomorphism is
related to the number, size, and topography of the lesions.
Parenchymal disease (as in this case) is most common and
presents with seizures in 50 to 80% of patients. Treatment
typically includes antiepileptics, albendazole, or praziquantel , as well as a short course of steroids to reduce the
inflammatory reaction during antihelminthic treatment.
Niclosamide may be given orally to treat adult tapeworms
in the GI tract. Fluconazole is an antifungal and not used to
treat this disease process. The colonization of the ventricular
system often presents with rapid clinical deterioration due
to increased intracranial pressure from obstructive hydrocephalus. There is still controversy about the best treatment
for this form of the disease, but most authors advocate either
a trial of antihelminthic medication, endoscopic cyst resection, or microsurgery. Subarachnoid disease is usually more
difficult to manage because the cysts are usually multiple,
attain larger sizes, and produce severe basal meningitis, but
antihelminthic medications are typically first-line therapy
for this form of the disease. From the many tests performed,
current data indicate that enzyme-linked immunosorbent
assay (ELISA) and electroimmunotransfer blot (EITB) tests
are the most effective laboratory tests for diagnosis. Peripheral white blood cells, ova and parasites in stool, and eosinophilia are inconsistent and unreliable markers of disease.
This patient also harbored a fourth ventricular cyst and
obstructive hychocephalus, which accounts for the rounded
third ventricle (Greenberg, pp. 236- 239)

79
Q

This disorder is caused by

A. Borrelia bU”I-gd01:!’eri
B. Echi:llOCOCCUS gnm:ulosa
C. Toxoplasma gondi:i
D. Trepollema pa/lidum
E. Taenia soi’ium

A

A. Borrelia bU”I-gd01:!’eri
B. Echi:llOCOCCUS gnm:ulosa
C. Toxoplasma gondi:i
D. Trepollema pa/lidum
**E. Taenia soi’ium **

Neurocysticercosis (NCC) is the most common parasitic infection of the central nervous system (CNS)
worldwide. Humans are the definitive host for the adult tapeworm Taenia so/iwn, which thrives in the small intestine
without consequence. Fecal shedding of eggs usually leads to
ingestion of eggs in contaminated water or food by an intermediate host, typically humans or pig. Once inside the intestine, the eggs are released and produce primary larvae that
enter the Circulatory system. Hematogenous spread to muscular, ocular, and neural tissue then occurs. Once inside the
brain, the primary larvae develop into secondary larvae, the
cysticerci. Clinical manifestations of the neural form of
the disease are varied and nonspecific. This pleomorphism is
related to the number, size, and topography of the lesions.
Parenchymal disease (as in this case) is most common and
presents with seizures in 50 to 80% of patients. Treatment
typically includes antiepileptics, albendazole, or praziquantel , as well as a short course of steroids to reduce the
inflammatory reaction during antihelminthic treatment.
Niclosamide may be given orally to treat adult tapeworms
in the GI tract. Fluconazole is an antifungal and not used to
treat this disease process. The colonization of the ventricular
system often presents with rapid clinical deterioration due
to increased intracranial pressure from obstructive hydrocephalus. There is still controversy about the best treatment
for this form of the disease, but most authors advocate either
a trial of antihelminthic medication, endoscopic cyst resection, or microsurgery. Subarachnoid disease is usually more
difficult to manage because the cysts are usually multiple,
attain larger sizes, and produce severe basal meningitis, but
antihelminthic medications are typically first-line therapy
for this form of the disease. From the many tests performed,
current data indicate that enzyme-linked immunosorbent
assay (ELISA) and electroimmunotransfer blot (EITB) tests
are the most effective laboratory tests for diagnosis. Peripheral white blood cells, ova and parasites in stool, and eosinophilia are inconsistent and unreliable markers of disease.
This patient also harbored a fourth ventricular cyst and
obstructive hychocephalus, which accounts for the rounded
third ventricle (Greenberg, pp. 236- 239)

80
Q

What is the most Bllely diagnosis depicted by the
angiogram below (Figure 6.80Q)?

A. Blue rubber bleb nevus syndrome
B. Vein of Galen aneurysm
C. Carotid-cavernous fistula
D. Intracranial hemangioblastoma of infancy
E. Sinus pericranii

A

A. Blue rubber bleb nevus syndrome
**B. Vein of Galen aneurysm **
C. Carotid-cavernous fistula
D. Intracranial hemangioblastoma of infancy
E. Sinus pericranii

A neonate suffering high-output cardiac failure,
hyperdynamic precordium, dilated cervical and cranial
veins, and arteries with a “machine-like” bruit heard over
the head and neck is most likely harboring a vein of Galen
aneurysm. Vascular tumors, CC fistula, blue rubber bleb
nevus syndrome, and sinus pericranii do not typically present with this constellation of problems. Although transcranial ultrasonography is an excellent way to diagnose
these lesions, the “gold standard” is cerebral angiography. In
infants, a transarterial and transvenous route to eliminate
the high-flow shunt is often employed . This technique may
not lead to complete obliteration but often converts highoutput cardiac failure to a persistent fistula. In an older child
or adult, the treatment is frequently gradual and entails graded elimination of the shunt with endovascular surgery,
usually via a transarterial route (Kaye and Blacl<, pp. 174-
176; Youmans, pp. 3433-3445).

81
Q

The posterior interosseous nerve supplies all of the
following muscles EXCEPT?
A. Supinator
B. Extensor carpi ulnaris
C. Abductor pollicis longus
D. Extensor cligitorum
E. Pronator quadratus

A

A. Supinator
B. Extensor carpi ulnaris
C. Abductor pollicis longus
D. Extensor cligitorum
**E. Pronator quadratus **

The deep branch of the radial nerve passes through a
slit in the supinator muscle (arcade of Frohse) to the posterior forearm. rVter passing this slit, the nerve is called the
posterior interosseous nerve and supplies the supinator,
extensor carpi radialis breviS, extensor digitorum, extensor
digiti minimi, extensor carpi ulnaris, abductor pollicis
longus, extensors pollicis longus and breviS, and extensor
indicis muscles. The anterior interosseous nerve, a branch
of the median nerve, supplies the pronator quadratus
(Greenberg, pp. 523, 542- 543).

82
Q

The ability to create irregularly shaped radiosurgical
volumes is important to achieve conformal irradiation of
target tissue. Which of the following techniques can be
employed to create such plans?
A. Combine multiple isocenters of irradiation in different
planes
B. Individual isocenters can be weighted variably to
change their relative shape
C. Individual radiation beams can be blocked to restrict
dose away from critical structures, such as the optic
chiasm
D. A and B only
E. All of the above

A

A. Combine multiple isocenters of irradiation in different
planes
B. Individual isocenters can be weighted variably to
change their relative shape
C. Individual radiation beams can be blocked to restrict
dose away from critical structures, such as the optic
chiasm
D. A and B only
E. All of the above

The ability to create irregularly shaped radiosurgical
volumes is important to achieve conformal irradiation of
target tissue, as all tumors or lesions are rarely perfect
spheres. The follOWing techniques can be used to create an
irregularly shaped plan during radiosurgery. First, combine
multiple isocenters of irradiation in different planes. For
example, a series of 4-mm isocentors of irradiation is
often used to tailor radiation to the porus acusticus for
schwannomas. Second, individual isocenters can be weighted
variably to change their relative shape. Finally, individual
radiation beams can be blocked to restrict dose away from
critical structures, such as the optic chiasm (Youmans,
pp. 4118- 4119).

83
Q

A 45-year-old female undergoes a C5-6 and C6-7 anterior cervical discectomy and fusion. Postoperatively, she
awakens with a Horner’s syndrome. The most likely etiology
of this finding was related to damage of what structure(s) ?
A. Sympathetic nerves ruuning along the carotid artery
during neck dissection
B. Injury of the T1 nerve root during the discectomy
C. Interruption of the sympathetic chain located on the
anterior surface of the longus colli muscles
D. Spinal cord injury during surgery
E. A small hypothalamic infarct during surgery

A

A. Sympathetic nerves ruuning along the carotid artery
during neck dissection
B. Injury of the T1 nerve root during the discectomy
**C. Interruption of the sympathetic chain located on the **
anterior surface of the longus colli muscles
D. Spinal cord injury during surgery
E. A small hypothalamic infarct during surgery

A rare complication after anterior cervical procedures is the development of a Horner’s syndrome (anhidrosis,
miosis, ptosis) from interruption of the sympathetic chain
located on the anterior surface of the longus colli muscle.
The thoracic duct enters the subclavian vein on the left and
is particularly vulnerable to injury during left-sided anterior
cervical procedures (Youmans, pp. 4442- 4443, 4451).

84
Q

A left-sided approach decreases the risl( of recurrent
laryngeal nerve palsy during anterior cervical procedures,
but at lower levels in the neck a left-sided approach runs the
risk of injuring what structure?
A. Inferior laryngeal nerve
B. Thyrocervical artery
C. Thoracic duct
D. C5 nerve root
E. Dominant cardiac accelerator nerves

A

A. Inferior laryngeal nerve
B. Thyrocervical artery
**C. Thoracic duct **
D. C5 nerve root
E. Dominant cardiac accelerator nerves

A rare complication after anterior cervical procedures is the development of a Horner’s syndrome (anhidrosis,
miosis, ptosis) from interruption of the sympathetic chain
located on the anterior surface of the longus colli muscle.
The thoracic duct enters the subclavian vein on the left and
is particularly vulnerable to injury during left-sided anterior
cervical procedures (Youmans, pp. 4442- 4443, 4451).

85
Q

Degenerative spondylolisthesis is most common at what
level in the lumbar spine?
A. Ll-2
B. L2-3
C. L3-4
D. L4-5
E. L5-S1

A

A. Ll-2
B. L2-3
C. L3-4
**D. L4-5 **
E. L5-S1

Degenerative spondylolisthesis is most common in
women at the L4-5 level, but may also be seen at the L3-4
level. Because this often occurs with sacralization of L5 in
many patients, the facet degeneration may be explained as a
hypermobility syndrome (Youmans, pp. 4545-4546)

86
Q

A 72-year-old female with rheumatoid arthritis is found
to have a reducible atlantoaxial dislocation after 36 hours of
cervical traction. There is minimal ventral compression from
pannus formation, no cranial settling, and no foramen magnum stenosis noted on MR scan. What is the best treatment
strategy for this patient?
A. Transarticular screw fL’wtion and fusion if the lateral
atlantalmasses are intact with good-quality bone
B. Transoral odontectomy followed by posterior occipitalcervical decompression and fusion
C. Laminectomy
D. Transoral odontectomy followed by observation
E. Halo placement

A

**A. Transarticular screw fL’wtion and fusion if the lateral
atlantalmasses are intact with good-quality bone **
B. Transoral odontectomy followed by posterior occipitalcervical decompression and fusion
C. Laminectomy
D. Transoral odontectomy followed by observation
E. Halo placement

Irreducible pathologic conditions affecting the cervicomedullary junction in patients with rheumatoid arthritis
frequently require an anterior cervical procedure to remove
the offending pathology (frequently pannus formation), followed by dorsal fL”ation and fusion. Reducible atlantoaxial
dislocation is best managed by a posterior fusion and fL,,-
ation procedure. Usually transarticular screw fL”ation is
desirable if the bone quality is good and the lateral masses
of the atlas are intact; but if this is not the case, placement
of a rectangle of bone between the posterior arch of C2
and Cl followed by wiring may be an option. This, however,
has been associated with higher failure rates in some studies These patients often required further reinforcement with
cervical immobilization techniques such as halo placement.
Dorsal occipitocervical fusion is almost always required
for patients with rheumatoid cranial settling and in those
following rheumatoid pannus resection. Transoral odontectomy with or without posterior fusion is not required for
this patient since there was a reducible lesion. Laminectomy
alone would likely further destabilize this patient, while
halo placement without fusion would run a very high risk
of failure (Youmans, pp. 4578- 4550; Wili(ins, pp. 3789-
3790).

87
Q

The ideal bone graft provides all of the following elements for successful healing EXCEPT?
A. Osteoconductive matrb::
B. Osteoinductive factors
C. To support viable osteogenic cells
D. Structural support
E. Osteoblasts for bone healing

A

A. Osteoconductive matrb::
B. Osteoinductive factors
C. To support viable osteogenic cells
D. Structural support
E. Osteoblasts for bone healing

The ideal bone graft provides the following elements
for successful healing: osteoconductive matrix, osteoinductive factors, viable osteogenic cells, and structural support.
Only fresh autografts contribute viable osteogenic cells to
the developing fusion. Processed allografts frequently have
no living cellular elements and are mainly derived from the
tissues of the recipient bed (Youmans, pp. 4615-4616).

88
Q

Interfering with uptake of which ion into cells during
severe closed head injury has resulted in a significant clinical
benefit?
A. Ca 2+
B. Na+
C. CID. Ie
E. None of the above

A

A. Ca 2+
B. Na+
C. CID. Ie
E. None of the above

Although traumatic brain injury (TBI) has been
shown to result in increases of calcium flux into cells with
subsequent cell injury, no clinical benefit has been observed
in clinical trials attempting to attenuate this response in
patients with TBI. A subset of patients with subarachnoid
hemorrhage, however, did show a benefit. Calcium may
enter cells via ion channels influenced by excitatory amino
acids (glutamate, aspartate). Unfortunately clinical trials to
antagonize these receptors have been discouraging in TBI
(Youmans, pp. 5025- 5027).

89
Q

Surgical therapies used for dystonia have traditionally
included all of the following EXCEPT?
A. Peripheral denervation
B. Pallidotomy
C. Thalamotomy
D. Dorsal column stimulation
E. Motor cortex stimulation

A

A. Peripheral denervation
B. Pallidotomy
C. Thalamotomy
D. Dorsal column stimulation
E. Motor cortex stimulation

Surgical therapies including cerebellar stimulation,
dorsal column stimulation, peripheral denervation, thalamotomy, and pallidotomy have been used in the past to
treat various forms of dystonia. Although the tha lamus has
been the primary target for years, more recen tly many surgeons are targeting the globus pallidus with good results.
Medications such as anticholinergics, muscle relaxants, and
benzodiazepines are of limited use to patients. Botulism
toxin is a safe and effective therapy for many focal dystonias
but has not proven effective for patients with segmental
dystonia, hemidystonia, or generalized dystonia (Youmans,
pp. 2795-2801).

90
Q

All of the following surgical procedures have been
employed to treat neuropsychiatric illness and behavioral
disorders EXCEPT?
A. Arcuate fasciculotomy
B. Subcaudate tractotomy
C. Limbic leukotomy
D. Anterior capsulotomy
E. Anterior cil1gulotomy

A

**A. Arcuate fasciculotomy **
B. Subcaudate tractotomy
C. Limbic leukotomy
D. Anterior capsulotomy
E. Anterior cil1gulotomy

The surgical management of psychiatric disease
can be helpful for select patients with treatment-refractory
major affective disorders, obsessive-compulsive disorder,
and chronic anxiety states. Surgical interventions have
included anterior capsulotomy, limbic leukotomy, subcaudate tractotomy, and anterior cingulotomy but not arcuate
fasciculotomy (Youmans, pp. 2853-2862).

91
Q

The superior semicircular canal projects into the Hoor of
the middle cranial fossa as what structure often seen during a
subtemporal approach for acoustic neuroma resection?
A. Arcuate eminence
B. Tegmen tympani
C. Vestibule
D. Vertical crest
E. Vestibular prominence

A

**A. Arcuate eminence **
B. Tegmen tympani
C. Vestibule
D. Vertical crest
E. Vestibular prominence

The superior semicircular canal projects into the floor
of the middle cranial fossa as the arcuate eminence (Tew,
pp. 48- 49; Wili(ins, pp. 1071-1073).

92
Q

A 45-year old male undergoes a sub temporal
approach for tumor resection with elevation of the dura from
the middle fossa floor and petrous bone.

Structures visible on the Hoor of the middle cranial
fossa during this exposure may include all of the following
EXCEPT?
A. ~I’!icldle meningeal artery
B. Trigeminal nerve (V3)
C. Lesser superficial petrosal nerve
D. Hypoglossal nerve
E. Greater superficial petrosal nerve

A

A. ~I’!icldle meningeal artery
B. Trigeminal nerve (V3)
C. Lesser superficial petrosal nerve
**D. Hypoglossal nerve **
E. Greater superficial petrosal nerve

Structures often visible on the middle
fossa floor during subtemporal approach include the middle
meningeal artery (often sacrificed by cautery and packing
of the foramen spinosllm), trigeminal nerve (V3), lesser
superficial petrosal nerve, greater superficial petrosal nerve,
ICA (if there is a small dehiscence in the bone), as well as the
arcuate eminence, which overlies the superior semicircular
canal. Decreased tearing after surgery most likely resulted from
injury of the greater superficial petrosal nerve, which provides
parasympathetic supply to the lacrimal and nasal gland. Additional exposure to the posterior fossa during a sub temporal
approach may be gained by removing the bone of I<awase’s
quadrilateral located in the medial petrous apex, medial to
Glasscock’s triangle. I(awase’s quadrilateral is bounded laterally by the greater superficial petrosal nerve, medially by the
petrous ridge and V3 of the trigeminal nerve, and at its base
by the arcuate eminence. Glasscock’s triangle is bounded laterally by a line from the foramen spinosum to the facial hiatus,
medially by the GSPN, and at its base by the mandibular
division of the trigeminal nerve (Tew, pp. 48- 49, 385- 395)

93
Q

A 45-year old male undergoes a sub temporal
approach for tumor resection with elevation of the dura from
the middle fossa floor and petrous bone.

Postoperatively, the patient has decreased lacrimation
on the ipsilateral side. What is the most likely etiology of this
problem?
A. Lesser petrosal nerve injury
B. Greater petrosal nerve injury
C. Geniculate ganglion injury
D. Chorda tympani injury
E. Injury of Jacobson’s nerve

A

A. Lesser petrosal nerve injury
**B. Greater petrosal nerve injury **
C. Geniculate ganglion injury
D. Chorda tympani injury
E. Injury of Jacobson’s nerve

Structures often visible on the middle
fossa floor during subtemporal approach include the middle
meningeal artery (often sacrificed by cautery and packing
of the foramen spinosllm), trigeminal nerve (V3), lesser
superficial petrosal nerve, greater superficial petrosal nerve,
ICA (if there is a small dehiscence in the bone), as well as the
arcuate eminence, which overlies the superior semicircular
canal. Decreased tearing after surgery most likely resulted from
injury of the greater superficial petrosal nerve, which provides
parasympathetic supply to the lacrimal and nasal gland. Additional exposure to the posterior fossa during a sub temporal
approach may be gained by removing the bone of I<awase’s
quadrilateral located in the medial petrous apex, medial to
Glasscock’s triangle. I(awase’s quadrilateral is bounded laterally by the greater superficial petrosal nerve, medially by the
petrous ridge and V3 of the trigeminal nerve, and at its base
by the arcuate eminence. Glasscock’s triangle is bounded laterally by a line from the foramen spinosum to the facial hiatus,
medially by the GSPN, and at its base by the mandibular
division of the trigeminal nerve (Tew, pp. 48- 49, 385- 395)

94
Q

A 45-year old male undergoes a sub temporal
approach for tumor resection with elevation of the dura from
the middle fossa floor and petrous bone.

During surgery, additional exposure is needed to access
the upper petroclival region for tumor resection. Which
maneuver may assist the surgeon in accomplishing this
task?
A. Further drilling of Glasscock’s triangle
B. Additional exposure through Kawase’s quadrilateral
C. Further drilling of the arcuate eminence
D. Identifying Trautmann’s triangle and exposing
medially to this landmark
E. Modifying the approach by utilizing a presigmoid
corridor

A

A. Further drilling of Glasscock’s triangle
**B. Additional exposure through Kawase’s quadrilateral **
C. Further drilling of the arcuate eminence
D. Identifying Trautmann’s triangle and exposing
medially to this landmark
E. Modifying the approach by utilizing a presigmoid
corridor

Structures often visible on the middle
fossa floor during subtemporal approach include the middle
meningeal artery (often sacrificed by cautery and packing
of the foramen spinosllm), trigeminal nerve (V3), lesser
superficial petrosal nerve, greater superficial petrosal nerve,
ICA (if there is a small dehiscence in the bone), as well as the
arcuate eminence, which overlies the superior semicircular
canal. Decreased tearing after surgery most likely resulted from
injury of the greater superficial petrosal nerve, which provides
parasympathetic supply to the lacrimal and nasal gland. Additional exposure to the posterior fossa during a sub temporal
approach may be gained by removing the bone of I<awase’s
quadrilateral located in the medial petrous apex, medial to
Glasscock’s triangle. I(awase’s quadrilateral is bounded laterally by the greater superficial petrosal nerve, medially by the
petrous ridge and V3 of the trigeminal nerve, and at its base
by the arcuate eminence. Glasscock’s triangle is bounded laterally by a line from the foramen spinosum to the facial hiatus,
medially by the GSPN, and at its base by the mandibular
division of the trigeminal nerve (Tew, pp. 48- 49, 385- 395)

95
Q

What is the most likely mechanism accounting for the
Cushing response?
A. Herniation of the cerebellar tonsils through the foramen
m agn um
B. Brainstem distortion
C. Large hemispheric insult
D. Hypoxia of the brainstem
E. Posterior fossa mass

A

A. Herniation of the cerebellar tonsils through the foramen
m agn um

B. Brainstem distortion
C. Large hemispheric insult
D. Hypoxia of the brainstem
E. Posterior fossa mass

The Cushing response consists of the triad of hypertension, bradycardia, and an irregular breathing pattern.
According to many authors, the most likely mechanism
accounting for this response is reduction in oxygenation in
an area just rostral to the medulla. For this reason it is also
called the ischemic response (Greenberg, p. 642; Committee
on Education in Neurological Surgery, pp. 58, 155).

96
Q

What is the 1110st cOl11mon clin ical manifestation of the
abnormality depicted on the angiogram below (Figure 6.96-
6.97Q) ?
A. Hoarseness
B. Dysphagia
C. Unilateral tongue atrophy
D. Palpable neck mass
E. Hypertension

A

A. Hoarseness
B. Dysphagia
C. Unilateral tongue atrophy
**D. Palpable neck mass **
E. Hypertension

The most common clinical presentation of carotid
body tUlUors is a palpable neck mass in the high cervical
region. Less commonly patients present with hoarseness,
dysphagia, and unilateral tongue atrophy and wealmess due
to the tumors’ proximity to the vagus and hypoglossal nerves.
These tumors are generally benign, although they do tend to
locally invade adjacent tissue, which can make their resection difficult. An evaluation of the endocrine system may
be warranted, especially in patients with hypertension and
tachycardia. Some patients may harbor a pheochromocytomalike lesion that secretes excess catecholamines. In such
patients, a.-adrenergic blockade must be started about 2
weeks preoperatively to control hypertension, tachycardia,
and the potential for arrhythmia. Preoperative planning is
critical in these patients to reduce comorbidity. Some may
require preoperative embolization to reduce the amount of
bleeding during surgery (Youmans, pp. 1677-1681).

97
Q

Clinicians must be aware of what endocrine comorbidity
in evaluating patients with this tumor?
A. Diabetes insipidus
B. Pheochromocytoma
C. I-Iyperprolactinemia
D. Phenylketonuria
E. None of the above

A

A. Diabetes insipidus
**B. Pheochromocytoma **
C. I-Iyperprolactinemia
D. Phenylketonuria
E. None of the above

The most common clinical presentation of carotid
body tUlUors is a palpable neck mass in the high cervical
region. Less commonly patients present with hoarseness,
dysphagia, and unilateral tongue atrophy and wealmess due
to the tumors’ proximity to the vagus and hypoglossal nerves.
These tumors are generally benign, although they do tend to
locally invade adjacent tissue, which can make their resection difficult. An evaluation of the endocrine system may
be warranted, especially in patients with hypertension and
tachycardia. Some patients may harbor a pheochromocytomalike lesion that secretes excess catecholamines. In such
patients, a.-adrenergic blockade must be started about 2
weeks preoperatively to control hypertension, tachycardia,
and the potential for arrhythmia. Preoperative planning is
critical in these patients to reduce comorbidity. Some may
require preoperative embolization to reduce the amount of
bleeding during surgery (Youmans, pp. 1677-1681).

98
Q

A 45-year-old male has a long history of epilepsy from
seizure foci originating in the right premo tor cortex and
extending into the adjacent motor cortex. His seizures have
remained refractory to a variety of antiepileptic drugs,
and he was referred to a neurosurgeon to discuss surgical
options. EEG recordings reveal a seizure focus in the left
premotor region that extends to the adjacent motor cortex .
Which of the following surgical procedures may be performed concomitantly during lesionectomy to avoid major
injury to the motor cortex and help control his seizures?
A. Topectomy
B. Limitedlesionectomy
C. Motor cortex stimulation
D. Multiple subpial transections
E. Vagal nerve stimulation

A

A. Topectomy
B. Limitedlesionectomy
C. Motor cortex stimulation
**D. Multiple subpial transections **
E. Vagal nerve stimulation

Multiple subpial transection (MST) was developed as
a procedure to address seizure activity that extends beyond
the area of resection and into eloquent cortex. The cerebral
cortex has functional vertical columns, with its vertical orientation of incoming and outgoing fibers. Seizures, however,
are believed to spread horizontally through the cortex. MST
involves disconnecting the vertical columns of the cerebral
cortex, which inhibits synchronization and spread of the
seizure focus with minimal injury to the cortex. The most
common problem faced by patients after this procedure
includes subtle, transient deficits corresponding to the area
of resection that typically improve. Permanen t complication
rates after this procedure are in the order of 5% (Youmans,
pp. 2635-2642).

99
Q

What is the most COlllmon neurologically related complication after vagal nerve stimulator placement?
A. Facialnumbness
B. Bradycardia
C. Dysphonia
D. IIypotension
E. Short-lived arrhythmia

A

A. Facialnumbness
B. Bradycardia
**C. Dysphonia **
D. IIypotension
E. Short-lived arrhythmia

The most C0I111110n surgical complication after vagal
nerve stimulator (VNS) placement is infection. Transient vocal
cord paralysis with hoarseness and swallowing problems
is the second 1110st C0111mon surgical complication of VNS.
Temporary lower face numbness and wealmess occur in about 0.7% of patients, likely related to high cervical incisions and superficial nerve injury (Youmans, pp. 2649-2650).

100
Q

Which of the following statements concerning stabilization of the lumbar spine with segmental pedicle screw
iL,ation is correct?
A. The lateral stability is significantly enhanced if the
pedicle screw angle is 30 degrees or greater
B. The use of transiL,ation increases the rotational but
not the lateral load stability of the construct
C. Without a transfixator, the vertebral column is stable
in lateral load
D. None of the above
E. All of the above

A

**A. The lateral stability is significantly enhanced if the
pedicle screw angle is 30 degrees or greater **
B. The use of transiL,ation increases the rotational but
not the lateral load stability of the construct
C. Without a transfixator, the vertebral column is stable
in lateral load
D. None of the above
E. All of the above

The internal stabilization of two adjacent segments
of the lumbar spine with a pedicle screw construct having a
pedicle-to-pedicle screw angle of zero and no transfi…xitor is
not stable in lateral or rotational load, as each of the screws
are free to turn in their screw holes in the body. Stability can
be enhanced by the application of a transfi…xitor or angling
the screws inward to form a pedicle-to-pedicle screw angle of
30 degrees (Carson, pp. 893- 901; Committee on Education in
Neurological Surgery, pp. 36, 128).